Vous êtes sur la page 1sur 55

in CIVIL LAW

Prepared by: Atty. Roney Jone P. Gandeza

REVIEW NOTES IN was not expressed in accordance with the


formalities of wills, the donation is void.
CIVIL LAW
2. Rey drafted and properly executed a
ATTY. RONEY JONE P. GANDEZA notarial will. Assume the following clauses in
UNIT 10 2nd FLOOR BBCCC BLDG. his will and the following events:
NO. 56 COOPERATIVE ST. COR. ASSUMPTION
RD. (a) Reys will provides, I leave my 2011
BAGUIO CITY white Mitsubishi Montero with Plate No.
__________ AFB 346 to my friend Abe. At the time of
Reys death, Abe has already died, leaving
LAW PROFESSOR one child, Zandro.
COLLEGE OF LAW
UNIVERSITY OF THE CORDILLERAS (b) Reys will provides, I leave to my
HARRISON ROAD, BAGUIO CITY friend, Abe, my 2011 white Mitsubishi
__________ Montero with Plate No. AFB 346. Just prior
to Reys death, he sold the Montero.
1. Abe is to be operated on for a
cancerous tumor. Before the operation, he Discuss fully each situation and the
delivers an envelope to his friend, Rey. The legal effect of the events on the
envelope contains a letter saying, I realize testamentary provisions.
my days are numbered, and I want to give
you this check for P1 million in the event of (a) The bequest to Abe is rendered void
my death from this operation. Rey cashes by Abes predecease. The property devolves
the check. to Reys heirs in intestacy. Zandro, Abes son,
has no right to the property because a
The surgeon performs the operation voluntary heir, such as Abe, who dies before
and removes the tumor. Abe recovers fully. the testator transmits nothing to his heirs.
Several months later, Abe dies from a heart (par. 1, Art. 856, CC)
attack that is totally unrelated to the
operation. Upon Abes death, the (b) The sale by Rey of the property
administrator of his estate immediately bequeathed to Abe constitutes a revocation
commences an action to recover from Rey of the bequest in favor of Abe. The law grants
the P1 million. Rey this right because of the ambulatory
character of a will. Abe has no right to
Will the suit prosper? question the sale because the right of an
instituted heir (legatee or devisee) is merely
Yes. The donation to Rey is a donation inchoate.
mortis causa, not inter vivos. It is a mortis
causa donation because Abe intended it to 3. Debtor owed Creditor P400, 000. The
take effect upon his death, i.e. in the event debt is the subject of a lawsuit, and the court
of my death from this operation. awards Creditor a judgment of P400, 000
against Debtor. To satisfy the judgment, the
That Abe died from a cause unrelated sheriff levies on Debtors family home in
to the operation does not detract from the Baguio City valued at P500, 000. Debtor
fact that Abes death is the operative cause opposes the levy on the allegation that the
that would have conveyed ownership of the family home is exempt from execution.
Php1 million given to Rey. Since the donation Judgment for whom?

BAR OPERATIONS 2013


I press toward the mark for the
prize of the high calling of God in
Christ Jesus. - Philippians 3:14

BARRISTERS CLUB Page 1 of 55


in CIVIL LAW

Prepared by: Atty. Roney Jone P. Gandeza

at will. Hence, this type of agency is


Judgment for Creditor. Under Article irrevocable.
160 of the Family Code, if judgment is
rendered against the owner or owners of a 6. Aristotle needs P100, 000. Socrates
family home, and the creditor has reasonable agrees to lend him the money, but not
ground to believe that the value of the family without security. Consequently, Aristotle
home is in excess of P300, 000 or P200, 000, delivers some of his jewelry to Socrates and
as the case may be, the creditor may apply for signs a power of attorney giving Socrates the
an order directing the sale of the family power, in case he fails to repay the loan, to
home. sell the jewelry as his agent for the best price
that can be obtained and to pay out of the
In the case presented, the value of proceeds the unpaid amount of the loan,
the Debtors family home is in excess of P300, giving any surplus to him. Having obtained
000, it being located in an urban area; hence, the money, Aristotle later tells Socrates that
the same may be sold at public auction to he revokes the power to sell.
satisfy the judgment against him.
Is the agency revoked?
The foregoing rule applies even if the
increase in value of Debtors family home No. Under the law of agency, the
resulted from improvements introduced by power is not revoked. Aristotle has no right to
Debtor. terminate the agency at will. This is because
the agency is coupled with an interest. If
4. Baldomero, a Filipino, marries Aristotle dies, the power is still not affected.
Antonette, his French girlfriend, in Paris. At
the time of the marriage, Baldomero is 7. A and B orally contract for the sale of
psychologically incapacitated to enter into As house for P1 million. A writes B a letter
marriage under Philippine law. Under French confirming the sale by identifying the parties
law, the marriage is voidable. Is the marriage and the essential terms description and
also voidable in the Philippines? location of the property, price, place of
payment, and method of payment and
The marriage is void. The rule is: signs the letter.
valid there, valid here; void there, void
here; voidable there, voidable here. Is the sale enforceable by court
However, the case falls under one of the action?
exceptions, Article 36 of the Family Code
which refers to a void marriage by reason of No. the sale was made orally.
the psychological incapacity of a spouse. However, A has made a written
memorandum of the oral land contract.
5. What is an agency coupled with an Because A signed the letter, he can be held to
interest? the oral contract by B. Because B has not
signed a written contract or memorandum, B
An agency coupled with an interest is can plead the Statute of Frauds as a defense,
a relationship created for the benefit of the and A cannot enforce the contract against
agent. The agent actually acquires a beneficial him.
interest in the subject matter of the agency.
Under these circumstances, it is not equitable 8. Seller, in reply to an inquiry from
to permit a principal to terminate the agency Buyer, sent a letter dated December 8
stating terms upon which he would sell 100

BAR OPERATIONS 2013


I press toward the mark for the
prize of the high calling of God in
Christ Jesus. - Philippians 3:14

BARRISTERS CLUB Page 2 of 55


in CIVIL LAW

Prepared by: Atty. Roney Jone P. Gandeza

to 300 computer units of a certain brand at a 10. A politician promises a newspaper


certain price. On December 16, Buyer sent a editor that he will pay the editor P50, 000 if
telegram to Seller ordering 90 computer the editor will publish a false story indicating
units on those terms. On December 18, Seller that the politicians opponent is gay.
sent a telegram to Buyer rejecting the order. Suppose the editor publishes the story, is he
The next day Buyer sent Seller a telegram entitled to collect the amount promised?
stating, Please enter an order for 150
computer units as per your letter of No, because the agreement is
December 8. Seller refused the order, and contrary to public policy. Considering that the
Buyer sued for breach of contract. Judgment parties are in pari delicto, the law will afford
for whom? them no relief against each other. The editor
may even be held criminally and civilly liable
Judgment for Seller. Buyers telegram for the libelous article.
of December 16, refusing to the terms stated
in Sellers letter of December 8, varied the 11. The Statute of Frauds:
number of computer units, and was therefore
a counter-offer. A counter-offer is in law a a. Mandates that both parties sign the
rejection of the original offer. On December written contract or note or
8, the Seller declined to fulfill the Buyers memorandum evidencing their
order, thus the negotiations between the two agreement.
parties was closed. The Buyers attempt to fall
back on the Sellers original offer by the b. Applies to all contracts under which
telegram of December 19, therefore, created payment of P500.00 is to be made.
no rights against the Seller.
9. A and B, Filipinos, both women, c. Has no requirement that the
enter into a same-sex marriage in Denmark, agreement of the parties be reduced
where such a marriage is valid. Is the to one document.
marriage also valid here?
d. Sets forth the elements of fraud as
The general rule enunciated in the applicable to inducing another party
first paragraph of Article 26 of the Family to enter into a contractual
Code should apply, under which marriages relationship.
valid where celebrated are also valid here.
The case does not fit into any of the (C) is correct. The Statute of Frauds has
exceptions enunciated in the foregoing no requirement that the parties agreement
provision of Article 26 of the Family Code. be reduced to one document. The Statute
Therefore, same sex marriage is valid here if requires the essential terms of certain
valid where celebrated. contracts to be evidenced by a writing, a note
or memorandum to be enforceable. (Art.
However, it would seem that the case 1403, CC) Any writing will suffice as long as it
falls under one of the exceptions to the contains every essential term of the oral
application of the proper foreign law i.e., it agreement that is supports. The writing need
runs counter to an important public policy of not be single document.
the forum marriage should be between a
man and a woman. Because of this important (A) is incorrect because the Statute of
public policy, same sex marriage is void here. Frauds requires only that the party to be
charged sign the written agreement or note
or memorandum. The signature requirement

BAR OPERATIONS 2013


I press toward the mark for the
prize of the high calling of God in
Christ Jesus. - Philippians 3:14

BARRISTERS CLUB Page 3 of 55


in CIVIL LAW

Prepared by: Atty. Roney Jone P. Gandeza

of the Statute of Frauds is met if the writing 14. Abe took his car to a carwash station
contains the signature of the party to be and asked to have it washed. While it is
charged or that of his agent. Therefore, both being washed, Abe went to a nearby mall for
parties do not need to sign the writing to be two hours. In the meantime, one of the
enforceable under the Statute. workers at the car wash had mistakenly
hand-waxed the car. When Abe came back,
(B) is incorrect because the P500.00 he was presented with a bill for a wash job
threshold under the Statute of Frauds is and a hand wax. Is Abe liable to pay for the
relevant only to contracts for the sale of hand wax job?
goods. (Art. 1403, 2(d), CC) Thus, a contract
for services is not within the Statute even if No. the doctrine of quasi-contract
the consideration is P500.00 or more, as long does not apply when there is a contract
as it can be performed in a year. between the parties. Although there was a
benefit bestowed to Abe, Abe did not receive
(D) is incorrect because the Statute of an unjust benefit because the hand wax job
Frauds contains no such provisions. The was not contemplated by his agreement with
Statute of Frauds requires certain agreements the owner of the car wash station.
to be evidenced by a writing signed by the
party sought to be bound. Despite its name, 15. Article 58 of the Family Code
the Statute does not set forth the elements of expressly provides that an action for legal
fraud as applicable to contractual separation shall in no case be tried before six
relationships. months shall have elapsed since the filing of
the petition. Is this requirement absolute?
12. Why does the law impose an obligation
in the case of quasi-contract? The requirement is not absolute. As a
rule, the filing of an action for legal separation
To prevent unjust enrichment. In an requires a cooling off period for six months.
action for unjust enrichment, recovery is (Art. 58, FC) However, when the ground
based upon the universally recognized moral alleged os one of those falling under RA 9262,
principle that one who has received a benefit also known as the Anti-Violence Against
has the duty to make a restitution when to Women and their Children Act of 2004, there
retain such benefit would be unjust. is no such cooling off period because the
courts are mandated to proceed with the
13. Pedro was seriously in a vehicular hearing of the case as soon as possible. (Sec.
accident. A bystander called Dr. Garcia to 19, Ra 9262)
render medical treatment while Pedro was
unconscious. Dr. Garcia sent Pedro a bill for 16. A and B are Filipino overseas workers
the reasonable value of his medical services. in Syria. They fell in love and decided to get
Pedro refuses to pay. Judgment for whom? married in accordance with Syrian law. On
the day of the wedding, A fell ill and could
Judgment for Dr. Garcia on the basis not make it to the ceremony. Upon advice of
of a quasi-contract. In the problem presented, his Syrian lawyer, he requested his best man
there was a benefit bestowed to Pedro, a to stand as proxy during the wedding. The
benefit which is compensable under the law; marriage was celebrated in accordance with
otherwise, Pedro will be unjustly enriched at Syrian law and valid there as such. Is the
the expense of Dr. Garcia. marriage valid here in the Philippines?

BAR OPERATIONS 2013


I press toward the mark for the
prize of the high calling of God in
Christ Jesus. - Philippians 3:14

BARRISTERS CLUB Page 4 of 55


in CIVIL LAW

Prepared by: Atty. Roney Jone P. Gandeza

Yes. Under Article 26 of the Family ascendants consist of one-half of the


Code, a marriage valid where celebrated is hereditary estates of their children and
valid in the Philippines. To this general rule, descendants. Pursuant to Articlr 893, if the
the law enumerates exceptions, but a testator leaves no legitimate descendants,
marriage by proxy is not one of those but leaves legitimate ascendants, the
enumerated. Hence, the marriage is valid. surviving spouse shall have a right to one-
Expressio unius est exclusion alterius. fourth of the hereditary estate which shall be
taken from the free portion.
17. H, a Filipino, marries W, an
American, in New York. At the time of the Applying the foregoing provisions,
marriage, H was psychologically Anastacia is therefore entitled to a legitime of
incapacitated to enter into marriage, P50, 000, while Magdalena is entitled to a
although the incapacity manifested itself five legitime of P25, 000 which is to be taken from
years after the wedding. Assuming that the the free portion. The remainder of the estate
marriage is valid in New York, is it also valid constitutes the free portion which goes to
here? Anastacia being the instituted heir.

No. The rule is:valid there, valid 20. X and Y orally agreed that X would
here, except those prohibited under Arts. 35 lease office space to Y at fixed rent as soon
(1), (4), (5) and (6), 36, 37 and 38 of the as the current lessee of the premises vacates
Family Code. The case falls squarely under in two years time. Almost two years later,
one of the exceptions enunciated in Article 26 and before he took possession of the
of the Family Code; namely, a void marriage premises, Y learned of a much more
under Article 36. advantageous opportunity and established
his office elsewhere. To compel Y to
18. Two second cousins, Filipinos, got perform, X brought suit to enforce the
married in Japan where such a marriage is agreement.
void. Is the marriage valid here?
Ys strongest argument in his suit
No. The rule is:valid there, valid would be:
here. Hence, void there, void here also.
This notwithstanding the fact that had the a. Mistake.
marriage been solemnized here, it would b. Impossibility of performance
have been perfectly valid. c. Statute of Frauds.
d. Autonomy of Contracts.
19. Abe is the legitimate son of the The Statute of Frauds is Ys strongest
widow, Anastacia. While single, Abe made argument. To be enforceable, the Statute
out a will naming his mother as sole heir to requires certain agreements to be evidenced
an estate valued at P100, 000. Later, Abe by a writing signed by the party to be
married Magdalena. charged, including agreements that can be
performed within one year from its making.
If Abe died while married to Because the lease in question is to be
Magdalena without changing his will, would performed after one year, the Statute of
his entire estate of P100, 000 go to his Frauds is Ys strongest argument.
mother, Anastacia? 21. Tiger, Phil, and Rory are co-owners of a
parcel of land. Tiger sells his undivided share
Article 889 of the Civil Code expressly in the property to Bubba. Phil later sells his
provides that legitimate parents or

BAR OPERATIONS 2013


I press toward the mark for the
prize of the high calling of God in
Christ Jesus. - Philippians 3:14

BARRISTERS CLUB Page 5 of 55


in CIVIL LAW

Prepared by: Atty. Roney Jone P. Gandeza

undivided share to Matt. Is bubba entitled to substituting the buyer in the enjoyment
redeem Phils share? thereof.
Yes, because the right of legal In the instant case, Kevin thus
redemption is not limited to the original co- acquired only the rights pertaining to the
owners. What matters is that the seller, Kobe, which is only one-third undivided
redemptioner (Bubba) was already a co- share of the co-owned lot.
owner at the time when another co-owner 25. Suppose Kobe sells the entire
(Phil) sells his undivided share. In the instant property to Kevin for P600, 000, what would
case, Bubba was already a co-owner when be the basis of the redemption price?
Phil sold his share. The redemption price should be one-
22. Suppose Phil, instead of a sale, third of the actual purchase price, (P200, 000)
donated his undivided share to Matt, may not of the prevailing market value of the
Bubba or Rory, or both redeem? property.
No, because the right of legal Thus, Article 1088 of the Civil Code
redemption may be exercised only if the provides that should any of the heirs sell his
share of a co-owner is alienated to a third hereditary rights to a stranger before the
person by onerous title. partition, any or all of the co-heirs may be
23. Suppose Rory later sells his share to subrogated to the rights of the purchaser by
Bubba, may Matt redeem? reimbursing him of the price of the sale,
No, because the right of legal provided they do so within the period of one
redemption may be exercised only if the month from the time they were notified in
share of a co-owner is alienated to a third writing of the sale by the vendor.
person by onerous title. 26. Testator T died with a will. He left no
The rationale behind the right of legal compulsory heirs and, consequently, was
redemption among co-owners is to reduce completely free to dispose of his properties,
the number of co-owners and avoid entry of without regard to legitimes. One of the
strangers into the co-ownership. If the clauses in his will perpetually prohibits the
alienation is in favor of a co-owner, the alienation or mortgage by his named
number of co-owners is already reduced and legatees or devisees of the properties
no stranger has entered the co-ownership. specified therein.
QUESTIONS 24-25 are based on the Is the clause valid?
following fact situation: No. The perpetual prohibition is valid
Kobe, Lebron and Dwyane are co- for only 20 years. The testamentary provision
owners of a parcel of land. Without the stipulated in Ts will prohibiting perpetual
knowledge of his co-owners, Kobe sells the alienation of mortgage of the properties
entire community property to Kevin. mentioned therein violated Articles 867 and
24. Is the sale valid? 870 of the Civil Code. Paragraph 4, Article
A co-owner who sells the whole 1013 of the same code which specifically
community property will affect only his own allows a perpetual trust in inapplicable.
share, and the transferee gets only what Article 1013 is among the Civil Code
would correspond to his grantor in the provisions on intestate succession, specifically
partition of the thing owned in common. on the State inheriting from a decedent, in
By virtue of the sale of the entire default of persons entitled to succeed. Under
community property by a co-owner, the buyer this article, the allowance for a permanent
thereby becomes a co-owner of the property trust, approved by a court of law, covers
to the extent of participation of the seller co- property inherited by the State by virtue of
owner since the sale produced the effect of intestate succession. The article does not cure

BAR OPERATIONS 2013


I press toward the mark for the
prize of the high calling of God in
Christ Jesus. - Philippians 3:14

BARRISTERS CLUB Page 6 of 55


in CIVIL LAW

Prepared by: Atty. Roney Jone P. Gandeza

a void testamentary provision which did not


institute an heir. 30. May E claim moral damages from
In the instant case, Ts estate cannot both vehicle owners?
be subjected indefinitely to a trust because
the ownership thereof would then effectively E can claim moral damages against
remain with him even in the afterlife. both vehicle owners because the rules on
(Orendain v. Estate of Rodriguez, June 30, damages arising from death due to a quasi-
2009) delict are also applicable to death of a
passenger caused by breach of contract by a
QUESTIONS 27-30 are based on the common carrier. (Arts. 1755, 1764, 2206 and
following fact situation: 2219, CC)

A passenger bus owned by A and a 31. To satisfy a final judgment rendered


cargo truck owned by B collided. C, a bus against H and W, husband and wife, the
passenger, suffered injuries, while D, sheriff levied on a house and lot constituted
another bus passenger, died. The drivers of by the couple as their family home. The
the two vehicles were at fault. C, the injured sheriff later sold the property at public
bus passenger, and E, the surviving heir of D, auction to C, the judgment creditor.
immediately filed suit against the owners of
both vehicles for damages. After some discussion, C agreed to
allow the couple to remain on the property
27. May A, the bus owner, successfully as lessees and to vacate the same in two
invoke due diligence in the selection and years. A year later, the couple brought an
supervision of his employees to avoid action to annul the auction sale on the
liability? allegation that the property is their family
home and its sale is void ab initio.
No, he cannot. This is so because his
liability as a common carrier is based on Will the action prosper?
breach of contract. Such a defense will only
serve to mitigate As liability because by then The action will not prosper. Even if
he will be considered as a debtor in good the subject property is a family home and,
faith. thus, should have been exempt from
execution, H and W should have asserted it as
28. May B, owner of the cargo truck, a family home and its being exempt from
invoke the same defense? execution at the time it was levied upon or
within reasonable time thereafter. Failure to
Yes, because his liability is based on a do so would estop the spouses from later
quasi-delict. claiming the exemption.

29. May C claim moral damages from It is evident that H and W did not
both vehicle owners? assert their claim of exemption within a
reasonable time. At no other time can the
C can claim moral damages against B, status of a residential house as a family home
owner of the cargo truck, because of the be set up and proved and its exemption from
injuries he suffered, but as against A, owner execution claimed but before the sale thereof
of the bus, C can claim moral damages only if at public auction.
he proves reckless negligence on the part of
the common carrier amounting to fraud.

BAR OPERATIONS 2013


I press toward the mark for the
prize of the high calling of God in
Christ Jesus. - Philippians 3:14

BARRISTERS CLUB Page 7 of 55


in CIVIL LAW

Prepared by: Atty. Roney Jone P. Gandeza

Settled is the rule that the right of A person whose occupation of a


exemption is a personal privilege granted to realty by sheer tolerance of its owner is not a
the judgment debtor and as such, it must be possessor in good faith; hence, not entitled to
claimed not by the sheriff but by the debtor the value of the improvements built thereon.
himself at the time of the levy or within a (Verno Padua-Hilario v. Court of Appeals,
reasonable time thereafter. (De Mesa v. et.al., January 19, 2000)
Acero, G.R. No. 185064, January 16, 2012)
34. S contracts to sell to B a parcel of
32. What is an easement of aqueduct? land. They agree that B shall pay the
Does the existence of an easement of right purchase price on October 25, and that in
of way necessarily include the easement of case of Bs failure to pay, the contract shall
aqueduct? be automatically rescinded. If B does not pay
on October 25, can he still pay on October
An easement of aqueduct is the right 29?
to make water flow thru intervening estates
in order that one may make use of said Yes, provided there has been no
waters. judicial or notarial demand for rescission of
the contract as of October 29. Under Article
The existence of the easement of 1592 of the Civil Code, in the sale of an
right of way does not necessarily include the immovable property, even though it may
easement of aqueduct. Consequently, the have been stipulated that upon failure to pay
right to dig trenches and to lay pipelines for the price at the time agreed upon the
the conducting of water is not included in a rescission of the contract shall of right take
contract granting a right of way (the rights place, the vendee may pay even after
given merely those of ingress and egress to expiration of the period, as long as no
and from the lot involved). demand for rescission of the contract has
been made upon him either judicially or by a
33. X promised to donate his friend, Y, a notarial act.
parcel of land. Relying on such promise, Y
constructed a house of strong materials on 35. In a contract to sell, is it necessary
the land. When X died, however, his son Z for the vendor to send a notarial rescission
inherited the land. In the suit filed by Z to when the vendee fails to pay the balance of
recover possession of the land from Y, the the purchase price?
latter invoked the right to be reimbursed of
his necessary and useful improvements on No. Rescission, whether judicially or
the allegation that he is a builder in good by notarial act, is not required to be done by
faith. the vendor. There can be no rescission of an
obligation that is still non-existing, the
Is Y a builder in good faith? suspensive condition not having happened.

No. The mere promise of X to donate In a contract to sell, there is no


the property to Y does not make the latter a contract to rescind, judicially or by a notarial
builder in good faith. This is because at the act, because from the moment the vendee
time when the improvement was built, such fails to pay on time the purchase price, the
promise was not yet fulfilled. It was therefore contract between the parties is deemed ipso
a mere possession by tolerance. facto rescinded.

BAR OPERATIONS 2013


I press toward the mark for the
prize of the high calling of God in
Christ Jesus. - Philippians 3:14

BARRISTERS CLUB Page 8 of 55


in CIVIL LAW

Prepared by: Atty. Roney Jone P. Gandeza

36. In a contract of sell, is the vendor


obliged to refund what the vendee paid
under the contract if the sale is not No. Article 1544 if the Civil Code on
consummated? double sales applies only where the same
thing is sold to different vendees by the same
Yes. The partial payment made by the vendor. It does not apply where the same
buyer must be returned to him, in the thing is sold to different vendees by different
absence of a stipulation regarding forfeiture vendors as in the case at bar.
of payments made by the parties. Such action
is but just and equitable under the premises. 38. Who between the two buyers is the
If it were otherwise, there will be unjust rightful owner of the lot?
enrichment on the part of the seller at the
expenses of the buyer. (Ordenv. Aurea, G.R. B is the rightful owner of the lot.
No. 172733 August 20, 2008) When A sold to B the property, ownership
thereof was transferred to B in accordance
37. When a co-owner seeks to exercise with Article 1496 of the Civil Code which
his right of legal redemption, within what provides that the ownership of the thing sold
period may he exercise such right? is acquired by the vendee from the moment it
is delivered to him in any of the ways
The right of legal redemption (or pre- specified in Articles 1497 to 1501.
emption) shall be exercised by a co-owner
within 30 days from written notice by the Article 1498, in turn, provides that
seller-co-owner. Actual knowledge when the sale is made through a public
notwithstanding, written notice is still instrument, the execution thereof shall be
required. equivalent to the delivery of the thing which
is the object of the contract, if from the deed
EXCEPTION: Actual knowledge by co-heirs or the contrary does not appear or cannot be
co-owners living in the same land with clearly inferred. In the problem presented,
purchaser; or co-owner was middleman in the sale between A and B contains nothing
sale to third party, the requirement of written contrary to intent to transfer ownership.
notice is dispensed with; period of
redemption begins to run from actual When A died in 1993, she no longer
knowledge. owned the lot, and, therefore, his brother
could not have inherited it. The Extrajudicial
QUESTIONS 37-39 are based on the Settlement with Simultaneous Sale did not
following fact situation: thus confer upon C ownership of the lot in
question; hence he could not have conveyed
A sold an unregistered land to B. it to D.
Upon As death, his brother C, executed an 39. Is the issue of Ds good faith relevant
extrajudicial settlement of As estate under in solving whether he has a preferential right
which he (C) adjudicated exclusively unto to the lot?
himself the lot and simultaneously sold the
same to D who was unaware of the prior The issue of good faith or bad faith is
sale. relevant only where the subject of the sale is
a registered land and the purchaser is buying
37. Is the rule on double sale under the same from the registered owner whose
Article 1544 of the Civil Code applicable to title to the land is clean. In such case the
the above problem? purchaser who relies on the clean title of the

BAR OPERATIONS 2013


I press toward the mark for the
prize of the high calling of God in
Christ Jesus. - Philippians 3:14

BARRISTERS CLUB Page 9 of 55


in CIVIL LAW

Prepared by: Atty. Roney Jone P. Gandeza

registered owner is protected if he is a bigamy. (Tenebro v. Court of


purchaser in good faith and fro value. (Ong v. Appeals, 424 SCRA 272)
Olasiman, 465 SCRA 464 [2006])
g) If the second marriage took place
40. Article 40 of the Family Code before the Family Code, Article 40
specifically provides that the absolute nullity does not apply. Consequently,
of a previous marriage may be invoked for there is no need for a judicial
purposes of remarriage on the basis solely of decree of nullity of the first
a final judgment declaring such previous marriage. (Ty v. Court of Appeals,
marriage void. What are the consequences 346 SCRA 86)
of this provision?
h) If the second marriage took place
a) Parties to a marriage are not after the Family Code, even if the
permitted to judge for first marriage took place before it,
themselves the nullity of their Article 40 applies because it has
marriage. Only the competent retroactive effect. (Atienza v. J.
courts have that authority. Brillantes, A.m. No. MTJ-92-706,
243 SCRA 32)
b) Prior to such declaration, the
validity of the first marriage is 41. S orally offers to sell his land to B for 300,
beyond question. 000. B accepts the offer and pays s the
purchase price. S in turn delivers to B the
c) A party who contracts a second title of the land. B now seeks to register the
marriage assumes the risk of land in his name but the Register of Deeds
being prosecuted for bigamy. refuses to register the sale in the absence of
a notarized deed of sale.
d) If the first marriage is void but a
party remarries without seeking Which of the following accurately
judicial nullity of his first describes Bs best remedy?
marriage, the second marriage is
also void for non-compliance with a) He may enter into the possession
Article 40 in relation to Articles 51 of the land as a buyer in good
to 53 of the Family Code. faith.

e) Bigamy is committed as long as b) He cannot compel S to return the


the first marriage, although has payment because the contract is
not been declared as such under unenforceable.
Article 40. (Mercado v. Tan, 337
SCRA 122) c) He may compel S to execute a
notarized deed of sale because
f) Bigamy is committed even if the the contract is valid and
second marriage is declared void enforceable.
on the ground of psychological
incapacity of one of the spouses d) He may sue S to return the
because such second marriage, purchase price because no one
although void, still produces legal may enrich himself at the
consequences, among which is expense of another.
incurring criminal liability for

BAR OPERATIONS 2013


I press toward the mark for the
prize of the high calling of God in
Christ Jesus. - Philippians 3:14

BARRISTERS CLUB Page 10 of 55


in CIVIL LAW

Prepared by: Atty. Roney Jone P. Gandeza

(C) is Bs best available remedy. The No, because in such a case, the third
notarized deed of sale is required demanded person-mortgagor (X), after paying the
by B is neither for validity nor for deficiency through foreclosure of the real
enforceability of the contract of sale. The sale estate mortgage, has the tight of a guarantor
is both valid and enforceable. The subject who can hold the vendee (z) liable for the
document is required by B only for his payment made, thus indirectly violating the
convenience; i.e., to allow him to register the prohibition under the law. (Art, 1484 (3), Civil
sale. Code)

42. Article 213 of the Family Code 44. Suppose in the preceding problem Y
enunciates the rule that no child below the assigns the promissory note to M, promising
age of seven years shall be separated from the latter that should Z default and the
the mother, except for compelling reasons. chattel mortgaged is foreclosed resulting in a
In custody cases, when should this provision deficiency, the assignor (Y) shall answer for
be applied by the courts at the time of the the deficiency to the assignee (M). Is this
filing of the petition for custody or at the stipulation valid?
time when the court is to decide who
between the parents is entitled to the Yes, because in such a case, it is no
custody of the child? longer the vendee (Z) who is held liable but the
vendor (Y). There is this no violation of the
The argument that the 7-year Recto Law that if the vendor avails himself of
reference in the law applied to the date when the right to foreclose, he is prohibited from
the custody case is filed, not the date when bringing an action against the purchase for the
the decision is rendered, is flawed. The unpaid balance.
matter of custody is permanent and
unalterable. If the parent who was given 45. What are the instances under the law
custody (either by law or by choice of the when form is an indispensable and
child) suffers a future character change and mandatory requirement for the validity of the
becomes unfit, the matter of custody can contract?
always be re-examined and adjusted.
(Espiritu, et.al., v. Court of Appeals., 242 SCRA Form is a mandatory requirement for
362 [1995]) the validity of the following contracts:

QUESTIONS 43-44 are based on the (1) If the value of the personal
following fact situation: property donated exceeds P5, 000,
the donation and the acceptance
X executes a real estate mortgage on shall be made in writing:
his land in favor of Y to answer for any otherwise, the donation is void.
deficiency that may result from foreclosure (par. 3, Art. 748, CC);
of the chattel mortgage constituted over the
car sold on installments by Y to Z. (2) Donation of an immovable,
regardless of value, must be in a
43. Assuming there is a deficiency after public instrument (Art 748, CC);
foreclosure of the chattel mortgage, may Y
foreclose the real estate mortgage (3) A contract of partnership is void
constituted on Xs land? whenever property is contributed
thereto, if an inventory of said
property is not made, signed by

BAR OPERATIONS 2013


I press toward the mark for the
prize of the high calling of God in
Christ Jesus. - Philippians 3:14

BARRISTERS CLUB Page 11 of 55


in CIVIL LAW

Prepared by: Atty. Roney Jone P. Gandeza

the parties, and attached to the The following are the parties who may
public instruments (Art. 1773, CC); commence an action for annulment of
marriage and the periods for the filing of such
(4) Sale of piece of land through an action:
agent (Art. 1847, CC);
a) LACK OF PARENTAL CONSENT:
(5) Antichresis (Art. 134, CC); and The minor should bring the action within five
years after attaining the age of 21. For the
(6) Payment of interest (Art. 1956, parent or guardian, the action must be brought
CC). at any time before such party reaches the age
of 21.
46. If a marriage is declared void, how
should the properties acquired by the spouses b) INSANITY: The sane spouse or
during the marriage be liquidated, person having legal charge of the insane
partitioned, and distributed? spouse must bring the action at any time
before the death of either party. The insane
If a marriage is declared void, the spouse must bring the action during a lucid
properties of the spouses should be owned in interval or after regaining sanity, also before
the concept of co-ownership. A void or the death of the other party. The reason in not
voidable marriage, regardless of its ground, providing for a 5-year period is the insanity
cannot be governed by the conjugal recurs.
partnership of gains nor by the absolute
community of property regime. In such a case, c) FRAUD: Injured party must
Article 147of the Family Code applies, except bring the action within five years after
when the marriage is declared void for being discovery of the fraud.
bigamous, in which case Article 148 governs.
d) FORCE, INTIMIDATION,
(RJPG: The above rules assumes UNDUE INFLUENCE: The injured party must
significance in light of Articles 102 (absolute bring the action within five years from the time
community or property) and 129 (conjugal the force, intimidation or undue influence
partnership) of the Family Code which disappeared or ceased.
commonly provide that the conjugal dwelling
shall be adjudicated to the spouse with whom e) PHYSICAL INCAPACITY: The
the majority of the common children choose to injured party must bring the action within five
remain. Applying the above rule, if a marriage years after the marriage.
is declared void, the conjugal dwelling shall be
partitioned not in accordance with Articles 102 f) SEXUALLY-TRANSMISSIBLE
and 129, but in accordance with Articles 147 DISEASE: The injured party must bring the
and 148 of the Family Code. This holds true action within five years after the marriage.
even if majority of the children choose to
remain with one parent.) 48. Under the Domestic Adoption Act,
may the adopting parent file for rescission of
47. In an action for annulment of the decree of adoption?
marriage, who are the parties who ay
commence the action and within what period No. The Domestic Adoption Act had
may the action be filed? already abrogated the right of the adopting
parent to rescind a decree of adoption.
Nevertheless, the adopting parent can always,

BAR OPERATIONS 2013


I press toward the mark for the
prize of the high calling of God in
Christ Jesus. - Philippians 3:14

BARRISTERS CLUB Page 12 of 55


in CIVIL LAW

Prepared by: Atty. Roney Jone P. Gandeza

for valid reasons, cause forfeiture of certain


benefits that would otherwise accrue to the ANOTHER ANSWER: As pointed out by
adopted. For instance, upon the grounds the Supreme Court in Obrecido, the reckoning
provided by law, an adopter may deny the point is not the citizenship of the parties at the
adopted his legitime and by a will, may exclude time of the celebration of the marriage, but
him from having a share in the disposable free their citizenship at the time a valid divorce is
portion. (Lahom v. Sibulo, GR No. 143989, July obtained abroad by alien spouse capacitating
14, 2003) the latter to remarry.

49. Andy and Betty, both eligible to marry QUESTION 50-51 are based on the
each other, cohabited as husband and wife following fact situation:
without the benefit of marriage. A few weeks
before Betty was to give birth, however, Andy William, an American, and Marissa, a
married Cora, an old maid. Heartbroken, Filipina, cohabited as husband and wife
Betty gave birth to Debby a few weeks after without the benefit of marriage. During their
Andys wedding. A month following Debbys cohabitation, the couple bought from
birth, Cora died in a car accident. Andy Mauricio a parcel of land in Baguio City.
reunited with Betty and immediately married Although the deed of sale was placed in the
her. Is Debby legitimated by the marriage of names of both William and Marissa as buyers,
her parents? the sale was registered in the name of
Marissa alone because William was
Yes. It is clear from the problem that disqualified to own real properties in the
although Debby was conceived and born Philippines. It is sufficiently established that
outside of wedlock of her parents, she was the funds used to buy the property came
conceived at the time when her parents, Andy solely from William, as Marissa has no
and Betty, were not disqualified by any sufficient source of income.
impediment to marry each other. The
reckoning point for legitimation is the period After their relationship has turned
of conception the child, not his birth, and the sour and the two went separate ways,
subsequent valid marriage of the parents. William sold all his rights and interests in the
property to Nicasio, a Filipino.
49. What is the new interpretation given
to the second paragraph of Article 26 of the When Nicasio tried to register the
Family Code? property in his name, he discovered that the
certificate of title is already registered in the
In Republic v. Orbecido III, 472 SCRA name of Marissa, and that it has already been
114 [2005], the Supreme Court made a novel mortgaged.
pronouncement that paragraph 2 of Article 26
of the Family Code should be interpreted to 50. If William is the true buyer of the
include cases involving parties who, at the time property, what is the effect of the registration
of the marriage, were Filipino citizens but later of the property in the name of Marissa?
one of them becomes a naturalized citizen of a
foreign country and obtains a divorce decree. The registration of the property in the
The Filipino spouse should likewise be allowed name of Marissa does not make her the owner
to remarry as if the other party were a of the property in question. It is settled that
foreigner at the time of the marriage. To rule registration is not a mode of acquiring
otherwise will be a sanction absurdity and ownership. It only means of confirming the
injustice. fact of its existence with notice to the whole

BAR OPERATIONS 2013


I press toward the mark for the
prize of the high calling of God in
Christ Jesus. - Philippians 3:14

BARRISTERS CLUB Page 13 of 55


in CIVIL LAW

Prepared by: Atty. Roney Jone P. Gandeza

world at large. Certificates of title are not a


source of right. The mere possession of a John, being an alien, is absolutely
certificate of title does not make the holder prohibited from acquiring public and private
the true owner of the property. Thus, the mere lands in the Philippines. Considering that
fact that Marissa has the title of the disputed Nieves appeared to be the designated vendee
property in her name does not necessarily, of the property leased, she acquired sole
conclusively, and absolutely make her the ownership thereto. This is true even of Johns
owner. claim that he provided the funds for such
acquisition is to be sustained. By entering into
51. Given that William is disqualified to such contract knowing that it was illegal, no
own real properties in the Philippines, is the implied trust was created in his favor; no
sale between him and Nicasio valid? reimbursement for his expenses can be
allowed; and no declaration can be made that
Given that aliens are disqualified to the subject property was part of the conjugal
own real properties in the Philippines, the or community property of the spouses. In any
transfer of the subject property from Mauricio event, he had and has no capacity or
to William, who is an American, would have personality to question the subsequent lease
been declared invalid if challenged, had not of the property by his wife in the theory that in
William conveyed the property to Nicasio who so doing, he was merely exercising the
is a Filipino citizen. It has been the consistent prerogative of a husband in respect of conjugal
ruling of the Supreme Court that if the land is property. To sustain such a theory would
invalidly transferred to an alien, who countenance indirect violation of the
subsequently becomes a Filipino citizen, or constitutional prohibition. If the property were
transfers it to a Filipino, the flaw in the original to be declared conjugal, as he would then have
transaction is considered cured and the title of a decisive vote as to its transfer or disposition.
the transferee is rendered valid. (Borromeo v. This is a right that the Constitution does not
Descallar, GR No. 159310, Febraury 24, 2009) permit him to have. Thus, the validity of the
lease must be upheld. (Mathews v. Taylor,
52. John, an American, and Nieves, a June 22, 2009)
Filipina, were married in Cebu City in 1997.
During their marriage, Nieves bought from 53. What are the recognized exceptions
Dionisia a beach lot in Boracay for P1.2 under the Constitution allowing aliens to own
million. The sale was financed by John. The private lands in the Philippines?
couple, also using Johns funds, introduced
improvement on the beach lot and eventually The instances when aliens are allowed
converted the property into a vacation and to acquire private lands in the Philippines are:
tourist resort. (a) By hereditary succession
(Section 7, Article XII, 1987 Constitution);
When the couple had a failing out,
Nieves leased the property to Matthews for a (b) A natural-born citizen of the
period of 25 years. She did so without Johns Philippines who has lost his Philippine
consent. citizenship may become a transferee of private
lands, subject to the limitations provided by
Does John have the legal standing to law. (Section 8, Article XII, Constitution).
question the validity of the lease agreement (Republic Act No. 8179 now allows a former
on the theory that in so doing, he was merely natural-born Filipino citizen to acquire up to
exercising his prerogative as a husband 5,000 square meter of urban land and 3
regarding conjugal property? hectares of rural land, and he may now use the

BAR OPERATIONS 2013


I press toward the mark for the
prize of the high calling of God in
Christ Jesus. - Philippians 3:14

BARRISTERS CLUB Page 14 of 55


in CIVIL LAW

Prepared by: Atty. Roney Jone P. Gandeza

land not only for residential purposes, but issued an original certificate of title over the
even for business or other purpose); and property in the name of Nicanor.

(c) Americans who may have Upon Vicentes death, Nicanors


acquired title to private lands during the brothers and sisters filed for judicial partition
effectivity of the Parity Agreement shall hold of the lot on the allegation that an implied
valid title thereto as against private persons trust over said property was created between
(Section 11, Article XII, 1973 Constitution). their brother Nicanor and their father
Vicente. Is this argument tenable?
54. Mao Tse Tung, a Chinese national,
bought from Juan dela Cruz, a Filipino, a The contention that an implied trust
parcel of land in Bulacan. Upon Maos death, was created between Nicanor and his father
the property was transferred by succession to does not hold water because the prohibition
his son, Chiang Kai Shek. When Chiang died, against an alien from owning lands of the
the property was in turn transferred by public domain is absolute and not even an
succession to his Filipino wife, Maria. implied trust can be permitted to arise on
equity considerations.
May the government successfully
prosecute a reversion case on the argument In the instant case, Nicanor became
that the sale of the land to Mao violated the the owner of the subject when he was granted
Constitutional provision disallowing aliens a miscellaneous sales patent by the Bureau of
from acquiring public and private lands in the Lands. Under the law, a certificate of title
Philippines? issued pursuant to any grant or patent
involving public land is a conclusive and
If the government had commenced indefeasible as any other certificate of title
reversion proceedings when the lot was still in issued to private lands in the ordinary or
the hands of Mao Tse Tung who was an alien cadastral registration proceeding.
disqualified to hold title thereto, the reversion
of the land to the State would undoubtedly be The effect of the registration of a
allowed. However, this is not the case here. patent and the issuance of a certificate of title
When the government instituted the reversion to the patentee is to vest in him an
case, the lot had already been transferred by incontestable title to the land, in the same
succession to Maria who is a Filipino citizen. manner as if ownership had been determined
And since the lot was transferred to a Filipino by final decree of the court, and the title so
citizen, the flaw in the original transaction is issued is absolutely conclusive and
considered cured. (Republic v. Register of indisputable, and is not subject to collateral
Deeds, GR 158230, July 16, 2008) attack, consequently, the lot is not part of
Vicente Tings estate. (Ting Ho, Jr. v. Teng Gui,
55. Vicente Ting, a Chinese national GR No. 130115, July 16, 2008)
married to a Filipina, occupied a public land in
Olongapo City. Pursuant to an affidavit, QUESTIONS 56-57 are based on the
Vicente transferred, without valuable following fact situation:
consideration, all his rights and interests over
the lot in favor of his eldest son, Nicanor. On Marcel and Monique, French
the basis of the affidavit, Nicanor, who earlier nationals, entered into a Contract to Sell with
obtained Filipino citizenship, was issued a Fil-Estate Realty Corp. for the purchase of a
miscellaneous sales patent by the Bureau of residential unit in a townhouse project. When
Lands. Not long after, the Register of Deeds Fil-Estate failed to comply with its verbal

BAR OPERATIONS 2013


I press toward the mark for the
prize of the high calling of God in
Christ Jesus. - Philippians 3:14

BARRISTERS CLUB Page 15 of 55


in CIVIL LAW

Prepared by: Atty. Roney Jone P. Gandeza

promise to complete the project at a certain Further, the spouses are not entitled to actual
date, the couple filed for rescission of the as well as interest thereon, moral and
contract. exemplary damages and attorney fees. (Hulst
v. PR Builders, Inc., 532 SCRA 41 [2007])
56. What it the status of the contract
entered into by Marcel and Monique with Fil- 58. H and W were married in 1978. In
Estate Realty Corp.? 1992, H obtained a decree of legal separation
after catching his wife having illicit relations
The Contract to Sell is void. Since with their neighbor. In the final decree of
Marcel and Monique, being French nationals, legal separation issued by the court, the court
are prescribed under the Constitution from ordered the forfeiture of Ws share in the net
acquiring and owning real property, it follows profits earned by the conjugal partnership in
that the Contract to Sell entered into by the favor of her children pursuant to Article 63(2)
parties is void. Under the Civil Code, all in relation to Article 129 of the Family Code.
contracts whose cause, object or purpose is
contrary to law or public policy and those Not satisfied with the ruling regarding
expressly prohibited or declared void by law the forfeiture of her share in the conjugal
are inexistent and void from the beginning. A assets, W claims that the net assets of the
void contract is equivalent to nothing; it conjugal partnership shall be computed in
produces no civil effect. accordance with Article 102 of the Family
Code (a provision under absolute community
57. In any event, are Marcel and Monique of property), instead of Article 129 (a
entitled to recover from Fil-Estate the amount provision on conjugal partnership). She
paid as well as interest and other damages? argues that Article 102 applies because there
are no other provisions under the Family
Since the contract involved here is a Code which defines net profits subject of
Contract to Sell, ownership has not yet forfeiture as a result of legal separation. She
transferred to Marcel and Monique when they contends that her vested right over half of the
filed the suit for rescission. While the intent to common properties of the conjugal
circumvent the constitutional prescription on partnership is violated when the forfeiture is
aliens owning real property was evident by to be made pursuant to Article 129 in relation
virtue of the execution of the Contract to Sell, to Article 63(2) of the Family Code.
such violation if the law did not materialize
because the spouses caused the rescission of Ws move raises the following issues:
the contract before the execution of the final
deed transferring ownership. a) What law governs the
property relations of the spouses
Thus, the exception to the application given that they were married before
of the pari delicto doctrine finds application in the effectivity of the Family Code?
the case at bar. Under the law, one who
repudiates the agreement and demands his b) What law governs the
money before the illegal act has taken place is dissolution of their common
entitled to recover. Marcel and Monique are properties since the decree of legal
therefore entitled to the recovery only of the separation was issued after the
purchase price. No damages may be recovered Family Code is already in effect?
on the basis of a void contract; being
nonexistent, the agreement produces no As to their property relations, the
juridical tie between the parties involved.

BAR OPERATIONS 2013


I press toward the mark for the
prize of the high calling of God in
Christ Jesus. - Philippians 3:14

BARRISTERS CLUB Page 16 of 55


in CIVIL LAW

Prepared by: Atty. Roney Jone P. Gandeza

Spouses are governed by the regime of community as a result of the liquidation and
conjugal partnership of gains. This is so settlement. The interest of each spouse is
because they were married when the limited to the net remainder resulting from the
operative law was the Civil Code. As to the liquidation of the affairs of the partnership
liquidation of their conjugal partnership assets, after its dissolution. Thus, the right of the
however, the Family Code is applicable husband or wife to one-half of the conjugal
because it is already the operative law at the assets does not vest until the dissolution of the
time of the dissolution of their conjugal marriage, when it is finally determined that,
partnership. after settlement of conjugal obligations, there
are net assets left which can be divided
In the instant case, the applicable law between the spouses or their respective
in so far as the liquidation of the conjugal heirs.
partnership assets and liabilities of H and W is
concerned is Article 129 of the Family Code 59. Is the computation of net profits
(liquidation of the conjugal partnership) in earned in the conjugal partnership of gains
relation to Article 63 (effects of a decree of the same as the computation of net profits
legal separation). The latter provision is earned in the absolute community?
applicable because insofar as Article 256 of the
Family Code [t]his code shall have retroactive The term net profits is defined in
effect insofar as it does not prejudice or impair Article 102(4) of the Family Code. Under this
vested or acquired rights in accordance with provision, the term net profits shall be the
the Civil Code or other law. increase in value between the market value of
the community property at the time of the
Ws contention that her vested right celebration of the marriage and the market
over half of the common properties of the value at the time of its dissolution. Without
conjugal partnership is violated when her any doubt, Article 102(4) applies to both the
share in the conjugal partnership is forfeited in dissolution of the absolute community regime
favor of her children pursuant to Article 63(2) under Article 102 of the Family Code, and to
and 129 of the Family Code has no basis. the dissolution of the conjugal partnership
regime under Article 129 of the Family Code.
While it is true that the couple were The difference lies in the process used under
married at the time when the operative law the dissolution of the absolute community
was the Civil Code, the Family Code should be regime under Article 102 of the Family Code,
given retroactive application for purposes of and in the processes used under the
determining the net profits earned by the dissolution of the conjugal partnership regime
conjugal partnership which is subject to under Article 129 of the Family Code.
forfeiture. A spouses claim of a vested right is
not etched in stone. To be vested, a right must ON ABSOLUTE COMMUNITY REGIME:
have become a title legal or equitable to Applying Article 102 of the Family Code, the
the present or future enjoyment of property. net profits requires a prior determination of
In one case, the Supreme Court reiterated its the market value of the properties at the time
long standing ruling that prior to the of the communitys dissolution. From the
liquidation of the conjugal partnership, the totality of the market value of all the
interest of each spouse in the conjugal assets is properties, the debts and obligations of the
inchoate, a mere expectancy, which absolute community are to be deducted and
constitutes neither a legal nor an equitable this will result to the net assets or net
estate, and does not ripen into title until it remainder of the properties of the absolute
appears that there are assets in the community, from which the value of the

BAR OPERATIONS 2013


I press toward the mark for the
prize of the high calling of God in
Christ Jesus. - Philippians 3:14

BARRISTERS CLUB Page 17 of 55


in CIVIL LAW

Prepared by: Atty. Roney Jone P. Gandeza

properties at the time of marriage is to be receive anything from the absolute


deducted, which then results to the net profits. community?

ON CONJUGAL PARTNERSHIP REGIME: If H and W have no separate


Applying Article 129 of the Family Code, the properties, the remaining properties of the
net profits requires a prior determination of couple are all part of the absolute community.
the separate properties and debts of the And its market value at the time of the
spouses under the following procedure a) an dissolution of the absolute community
inventory shall be prepared, listing separately constitutes the market value at dissolution.
all the properties of the conjugal partnership When H and W were legally separated, all the
and the exclusive properties of each spouse; b) properties which remained will be liable for
amounts advanced by the conjugal partnership the debts and obligations of the community.
in payments of personal debts and obligations Such debts and obligations will be subtracted
of either spouse shall be credited to the from the market value at dissolution. What
conjugal partnership as an asset thereof; c) remains after the debts and obligations have
each spouse shall be reimbursed for the use of been paid from the total assets of the absolute
his or her exclusive funds in the acquisition of community constitutes the net remainder or
property or for the value of his or her exclusive net asset. And from such net asset or net
property, the ownership of which has been remainder off the couples remaining
vested by law in the conjugal partnership; d) properties, the market value at the time of the
the debts and obligations of the conjugal marriage will be subtracted and the resulting
partnership shall be paid out of the conjugal totality constitutes the net profits. Since
assets. In case of insufficiency of said assets, both H and W have no separate properties,
the spouses shall be solidarily liable for the and nothing would be returned to each of
unpaid balance with their separate properties, them, what will be divided equally between
in accordance with the provisions of paragraph them are simply the net profits. However, the
2 of Article 121; e) whatever remains of the trial court forfeited the half-share of W in favor
exclusive properties of the spouses shall of her children. Thus, if Article 102 is used in
thereafter be delivered to each of them; f) the instant case (which should not be the
unless the owner had been indemnified from case), nothing is left to W since both parties
whatever source, the loss or deterioration of entered into their marriage without bringing
movables used for the benefit of the family, with them any property.
belonging to either spouse, even due to
fortuitous event, shall be paid to said spouse 61. Given that Article 129 of the Family
from the conjugal funds, if any; and g) the net Code applies to the liquidation of the conjugal
remainder of the conjugal partnership shall assets of H and W, is the latter entitled to
constitute the profits, which shall be divided receive any property from the conjugal
equally between husband and wife, unless a partnership?
different proportion or division was agreed
upon in the marriage settlements or unless No. What remains in the conjugal
there has been a voluntary waiver or forfeiture properties of H and W (after payment of all
of such share as provided in the Family Code. debts and obligations) should be divided
equally between the spouses. However, since
60. Suppose Article 102 of the Family W herself is the guilty party, her share from
Code (which is a provision under the regime the net profits of the conjugal partnership is
of absolute community of property) is to forfeited in favor of the common children
apply in the instant case, is W entitled to pursuant to Article 63(2) of the Family Code.
Nothing will be returned to W because in the

BAR OPERATIONS 2013


I press toward the mark for the
prize of the high calling of God in
Christ Jesus. - Philippians 3:14

BARRISTERS CLUB Page 18 of 55


in CIVIL LAW

Prepared by: Atty. Roney Jone P. Gandeza

conjugal partnership regime, there is no a) I accept your offer to sell the land.
separate property which may be accounted for I wish I could have gotten a better
in the guilty partys favor. (Quiao v. Quiao G.R. price.
No. 176556, July 4, 2012)
b) I accept your offer to sell the land,
62. S sells to B a retro a house and lot for but can you shave the price?
P2.5 million. The agreement of sale provides
that S cannot repurchase the property within c) I accept your offer to sell the land,
three years from the date of the contract. If but only if I can pay on 90 days
the sale is entered into in 2001, when can S credit.
repurchase the property?
d) I accept your offer to sell the land,
S can repurchase the property within provided that you are the owner.
four years from the expiration of the time
within which the right to redeem cannot be (C) is correct. Acceptance is a voluntary
exercised (here three years from date of the act by the offeree that shoes assent ot
contract). In other words, S can repurchase the agreement to the offer. The acceptance must
property until 2008 which is four years from be unequivocal and communicated to the
2004, the date of the expiration of the time offeror. The acceptance in (c) is not an
within which his right to redeem cannot be unequivocal acceptance because of the
exercised. This is pursuant to paragraph 1 of condition to pay on credit; such a condition
Article 1606 of the Civil Code which provides operates as a counter-offer.
that the right of the seller to repurchase the
property sold, in the absence of an agreement An acceptance may be unequivocal
as to the period of repurchase, shall last four even though the offeree expresses
years from the date of the contract. dissatisfaction with the offer, as in the case of
(a) and (b).
63. Suppose it is stipulated in the
agreement of sale that the vendor a retro can The condition in (d) that the offeror
repurchase the property whenever he or his owns the property is implied in every sale of
heirs have the means, when can S land, so the condition does not add any new or
repurchase? different terms to the offer.

S can repurchase the property within 65. A local civil registrar of a remote town
ten years from 2001; i.e., until 2010, if the in Ilocos Sur issued a marriage license on the
parties agree on the right to redeem without same day that the applicants filed their
specifying the period of redemption but from application. Will it affect the validity of the
the situation, facts or circumstances, it can be marriage?
inferred that the parties intended a period, the
vendor a retro may deem within ten years No, it will not. The failure of the local
from the date of the property sold within ten civil registrar to comply with the publication
years from the date of the contract. requirement under Article 17 of the family
Code is considered as a mere irregularity in a
64. S offers to B in a letter the sale of a formal requisite of marriage which will not
parcel of land. B sends a reply. Which of the affect the validity of the marriage, but would
following statements in Bs reply will not subject the guilty party administratively, civilly
result in a contract? or criminally liable.

BAR OPERATIONS 2013


I press toward the mark for the
prize of the high calling of God in
Christ Jesus. - Philippians 3:14

BARRISTERS CLUB Page 19 of 55


in CIVIL LAW

Prepared by: Atty. Roney Jone P. Gandeza

66. Cindy offers to sell to Daria a bleeding on the shoulder of the


particular car for P300, 000. Which of the highway. Z stops his car, gets out, and
following events will NOT terminate Cindys renders emergency medical care to
offer? the injured pedestrian.

a. A dies prior to Bs acceptance, and b) X, 82, asks her favorite niece, Y, a


at the time B accepts, he is unaware CPA, to fill out and file his income tax
of As death. return. Xs only income is from this
monthly pension from SSS and
b. The night before B accepts, fire interest income on a bank account. It
destroys the car. takes Y five minutes to complete the
form, takes a taxi, and goes to
c. B pays 1,000 for a 30-day option to personally file the return at the BIR.
buy the car. During this period, A dies,
and later A accepts the offer, knowing c) X, a contractor, has a contract to
of As death. paint Ys house. X mistakes the house
of Ys neighbor, Z, for Ys house. As X
d. A dies an hour before receiving Bs paints Zs house, Z stand by watching
acceptance. until the job is done.

(C) will not terminate Cindys offer. As d) A, a homeowner, has already paid
a rule, the death of either the offeror or his realty property tax. A clerk in the
offeree terminates the offer, except when the treasurers office mistakenly sends A
offer is irrevocable as in the case of an offer a bill that should have gone to As
founded upon an option. neighbor, B. Being a good citizen and
the thinking that the treasurers office
The event in (A) will terminate Cindys would not have sent him the bill if he
offer. The offerees power of acceptance is did not owe the money, A pays the
terminated when the offeror of offeree dies or bill.
is deprived of legal capacity to enter into the
proposed contract. An offer is personal to both An implied contract existed in (C). An
parties and cannot pass to the decedents heirs implied contract is a contract formed by
or assigns. manifestations of the parties other than oral or
written language, i.e., by conduct. In this case,
The event in (B) will automatically X, by beginning to paint Zs house, has made
terminate Cindys offer if the specific subject an offer by his conduct because a reasonable
matter of the offer is destroyed before the person would conclude that the services were
offer is accepted. offered with the expectation of compensation
rather than gratuitously. Xs offer has been
(D) will also terminate Cindys offer for accepted by Z even though the latter said
the same reason as in (A). nothing.

67. In which of the following fact The situation in (A) does not give rise
situations would a court most likely find that to an implied contract because the pedestrian
an implied contract existed? has not manifested his consent to the offer
made by X (in contrast to a patient who goes
a) X, a noted licensed physician, sees to Xs office and submits to treatment by hi,). X
an unconscious pedestrian lying is not without a remedy. He may recover the

BAR OPERATIONS 2013


I press toward the mark for the
prize of the high calling of God in
Christ Jesus. - Philippians 3:14

BARRISTERS CLUB Page 20 of 55


in CIVIL LAW

Prepared by: Atty. Roney Jone P. Gandeza

value of his services under a quasi-contract. A


quasi-contract is not really a contract at all; 69. Seller is obliged to deliver to Buyer
rather, it is a legal fiction designed to avoid one of his cars. Is the object of the sale
injustice by preventing unjust enrichment of determinate or indeterminate?
one party to the detriment of another.
The object of the sale refers to a class
The situation in (B) does not create an which in itself is determinate. Here, the
implied contract. The conduct of X and Y does particular thing to be delivered is
not appear from an objective standard to DETERMINABLE without the need of a new
manifest contractual intent because of the contract between the parties (Art. 1349, CC); it
close family relationship of the parties and the becomes determinate upon delivery.
minor burden on Y to render her services.
Courts generally will not presume that a 70. What is the mirror doctrine in the
contractual relationship was intended under law on sales?
these circumstances.
A purchaser of property cannot close
The situation in (D) likewise does not his eyes to facts which should put a reasonable
create an implied contract. Without some man on his guard and claim that he acted in
conduct on the part of B, such as knowingly good faith under the belief that there was no
accepting the offered benefits in silence, a defect on the vendors title.
court will not find the manifestation of mutual
consent necessary for an implied contract. The refusal to believe in the strong
possibility of a defect on the vendors title will
68. A man, prior to his marriage, made a not make the purchaser an innocent purchase
donation in a public instrument, in favor of for value, if circumstances are such that a
his future wife, on condition that should she reasonably prudent man would have taken the
die before him and there be no children, one- necessary precaution if in the same situation.
half of the properties donated shall be given (Embrado v. CA, 233 SCRA 333)
to the parents of his wife. Nine months after
the wedding, the wife died without issue. The 71. Apple Computers, Inc. and a
parents now claim the one-half share given to Philippine distributor entered into an
them in the deed of donation. Are the parents agreement whereby the distributor agreed to
entitled to the property? order 1,000 units of Apple Computers every
month and to resell them in the Philippines at
The parents cannot get the one-half the manufacturers suggested price plus 10%.
share. Insofar as said share is concerned, it All unsold units at the end of the year shall be
cannot be a valid donation propter nuptias nor bought back by the manufacturer at the same
a donation inter vivos nor a donation mortis price they were ordered. The manufacturer
causa. shall hold the distributor free and harmless
from any claim for defects in the unit.
The donation is not a donation propter
nuptias because the share was not given to Is the agreement one of sale or
one of the spouses. It is not a donation inter agency?
vivos, for there was no acceptance on the part
of the parents. It is not a donation mortis The contract is one of agency, not one
causa because the deed of donation did not of sale. Sale is negated by the following
have the formalities of a will, aside from the circumstances:
fact that the donor is still alive.

BAR OPERATIONS 2013


I press toward the mark for the
prize of the high calling of God in
Christ Jesus. - Philippians 3:14

BARRISTERS CLUB Page 21 of 55


in CIVIL LAW

Prepared by: Atty. Roney Jone P. Gandeza

the price is fixed by the manufacturer S. As additional security for the payment of
with the 10% mark-up constituting the the balance, C, a friend of B, executes a real
commission. estate mortgage over his land in favor of S.

the manufacturer reacquires the Is the Recto Law applicable if


unsold units at exactly the same price. additional security is given by a third person
who executes a real estate mortgage to
warranty for the units was borne by answer for a deficiency at foreclosure?
the manufacturer.
Yes, because in such a case, the third
The foregoing circumstances indicate a person-mortgagor (C), after paying the
sale because ownership of the units was never deficiency through foreclosure of the real
intended to transfer to the distributor. estate mortgage, has the right of a guarantor
who can hold the buyer (B) liable for the
72. Francisco donated to Cirila a parcel of payment made, thus indirectly violating the
land. The donation was made in a public Recto Law.
instrument, while the acceptance mad by
Cirila was embodied in the same public The Recto Law prohibits foreclosure
instrument. Upon the death of Francisco, his for such real estate mortgage because C would
nephews and nieces brought an action for the be entitled to proceed against B under the law
recovery of the property on the ground that on guaranty, thus resulting in indirect
the done, Cirila, was the common-law wife of circumvention of the prohibition.
their uncle at the time of the donation, and
that it is therefore coid under Article 87 of the 74. An agent is allowed to sell to himself
Family Code. Will the action prosper? what his principal has ordered him to buy. Is
this statement accurate?
Yes. Article 87 of the Family Code
expressly provides that every donation or That statement is inaccurate. The
grant of gratuitous advantage, direct or agent is allowed to sell to himself what his
indirect, between the spouses during the principal has ordered him to buy but only with
marriage shall be void, except moderate gifts the permission of his principal. If he does so
which the spouses may give each other on the without such permission, the sale is void.
occasion of any family rejoicing. The
prohibition shall also apply to persons living 75. S sells a piece of land to B who does
together as husband and wife without a valid not register the sale. Subsequently, S sells the
marriage. Since Francisco and Cirilia lived same piece of land to C who registers the
together as husband and wife without valid sale. In a suit between the buyers and B and C
marriage, the inescapable conclusion is that over the land sold, B (the first buyer) is
the donation made by Francisco in favor of defeated, and is deprived of the property.
Cirila is void under Article 87 of the Family
Code. (Arcaba v. Batocael, 370 SCRA 414 May B enforce against S the warranty
[2001]) against eviction?

73. B buys from S a car, payable in Yes. While it appears that B is evicted
installments. As security for the payment of by virtue of a right of subsequent to the sale to
the balance of the purchase price, a chattel him (not prior to the sale), the warranty may
mortgage is constituted on the car in favor of still be enforced because the cause of Bs
eviction is imputable to the vendor S.

BAR OPERATIONS 2013


I press toward the mark for the
prize of the high calling of God in
Christ Jesus. - Philippians 3:14

BARRISTERS CLUB Page 22 of 55


in CIVIL LAW

Prepared by: Atty. Roney Jone P. Gandeza

opportunity, declined Dr. Gos offer. A few


76. A offers to sell to B a particular car for weeks later, Dr. Lim wrote to Dr. Go saying
P300, 000. Which of the following events that he had reconsidered the matter and was
creates a contract between A and B? now accepting the offer previously made. Did
a contract arise?
a) A dies prior to Bs acceptance, and
at the time B accepts, he is unaware a) Yes, because Dr. Gos offer had no
of As death. termination date.

b) The night before B accepts, fire b) Yes, because Dr. Lim eventually
destroys the car. accepted the offer in writing.

c) B pays 1,000 for a 30-day option to c) No, because Dr. Lims rejection
buy the car. During this period, A dies, terminated the offer.
and later B accepts the offer, knowing
of As death. d) No, because Dr. Lims refusal
constitutes a counter-offer.
d) A dies an hour before receiving Bs
acceptance. (C) is correct because Dr. Lims
rejection terminated Dr. Gos offer. Dr, Lims
The event in (C) creates a contract belated acceptance of the offer is not the
between A and B because of the option money acceptance of an offer contemplated by law.
which continues to be effective despite the Such belated acceptance of Dr. Lim partakes of
offerors death, and despite the offerees an offer by him to Dr. Go which the latter is
knowledge of such death. free to accept or not.

77. S sold a retro to B a parcel of land. 79. A friend offered to sell Henry a laptop for
Within the period stipulated for redemption, P10,000. Henry replied, saying, Ill pay you
S failed to redeem. To register in the Registry P6,000 now and give you the balance at the
of Property his consolidation of ownership, B end of the month. Did a contract arise?
filed a petition for consolidation, but did not
name S as respondent. Consequently, S was a) Yes, because Henrys acceptance of
not duly summoned and heard. the offer was unconditional.

Has the court acquired jurisdiction? b) Yes, because the conditions added
by Henry merely involves the manner
No. S should have been named as a of payment of the purchase price.
respondent in the case, should have been
summoned, and should have been heard. The c) No, because the breach of the
requirement under Article 1607 of the Civil obligation is substantial enough that
Code for a judicial order is not required for would invalidate the consent of the
consolidation of ownership of the property, offeror.
but for purposes of registering the
consolidation of title. d) No because the conditions of
payment added by Henry constitutes
78. Dr. Go offered a partnership in his a counter-offer, thus invalidating the
private practice to Dr. Lim, a young surgeon. original offer.
Dr, Lim, hoping to secure a more favorable

BAR OPERATIONS 2013


I press toward the mark for the
prize of the high calling of God in
Christ Jesus. - Philippians 3:14

BARRISTERS CLUB Page 23 of 55


in CIVIL LAW

Prepared by: Atty. Roney Jone P. Gandeza

(D) is correct because the conditions of The rules regarding the liquidation of
payment added by Henry constitute a counter- the absolute community or conjugal
offer, thus invalidating the original offer. partnership are the same. They are as follows:

80. What are the steps to be undertaken 1. If a special proceeding for the
by the creditor in case of non-payment of the settlement of estate of deceased
debt on the things pledged? persons under the Rules of Court has
been instituted after the death of one
The steps are: spouse, the absolute community or
conjugal partnership shall be
(a) The creditor proceeds to a Notary liquidated in the said proceeding.
Public for the sale of the thing
pledged; 2. If no special proceeding for the
settlement of estate of the deceased
(b) The sale shall be made at a public spouse is instituted, the surviving
auction; spouse shall liquidate the absolute
community or conjugal partnership
(c) The creditor notifies the debtor and either judiciary or extra-judiciary
the owner of the thing pledged of the within one year from the death of the
public auction stating the amount for spouse.
which the public sale is made;
3. If no liquidation is made within one
(d) If at the first auction the thing is year from the death of the deceased
not sold, a second one with the same spouse, any disposition or
formalities shall be held; encumbrance involving any
community or conjugal property of the
(e) If at the second auction there is still terminated marriage shall be void.
no sale, the creditor may appropriate
the thing pledged and give an 4. Should the surviving spouse contract
acquittance of his entire claim. (Art. a subsequent marriage without
2112, CC) liquidating the community property or
conjugal partnership, a mandatory
81. What is the basis of payment of an regime of complete separation of
obligation in case of extraordinary inflation? property shall govern the property
relations of the subsequent marriage.
Extraordinary inflation exists when This is to protect the heirs of the
there is a decrease or increase in the deceased spouse. (Art. 103 and 130
purchasing power of the Philippine currency, FC)
and such increase or decrease could not have
been reasonable foreseen or was manifestly 83. When are the effects of extraordinary
beyond the contemplation of the parties at the inflation applicable?
time of the establishment of the obligation.
The effects of extraordinary inflation
82. What are the rules for the liquidation are applicable only where there is an official
of the absolute community of property or declaration to that effect by competent
conjugal partnership of gains in case of death authorities.
of a spouse?

BAR OPERATIONS 2013


I press toward the mark for the
prize of the high calling of God in
Christ Jesus. - Philippians 3:14

BARRISTERS CLUB Page 24 of 55


in CIVIL LAW

Prepared by: Atty. Roney Jone P. Gandeza

QUESTIONS 84-85 are based on the whom the actual owner of the vehicle is, the
following fact situation: operator of record continues to be the
operator of the vehicle as regards the public
Mayor Alfonso Favis of Magsingal, and third persons, and as such is directly and
Ilocos Sur was on board a Toyota Fortuner primarily responsible for the consequences
when the vehicle, cruising at high speed, incident to its operation. (Jayme v. Apostol,
accidentally hit the minor, Marvin, along the G.R. No. 163609, November 27, 2008)
national highway in Vigan City. Despite
medical attention, Marvin died six days after 86. For damage or injuries arising out of
the accident. negligence in the operation of a motor
vehicle, what is the nature of the liability of
The vehicle was then driven by Rodel, the registered owner?
an employee of the municipal government of
Magsingal, and registered in the name of For damage or injuries arising out of
Virgilio, a friend of Mayor Favis. negligence in the operation of a motor vehicle,
the registered owner may be held civilly liable
84. Is Mayor Favis solidarily liable for the with the negligent driver either:
negligence of the driver?
1) subsidiarily, if the aggrieved party
Mayor Favis is not liable. The doctrine seeks relief based on a delict or crime
of vicarious liability or imputed liability finds under Articles 100 and 103 of the
no application in the present case. Mayor Favis Revised Penal Code; or
was neither the employer of Rodel nor the
vehicles registered owner. There existed no 2) solidarily, if the complainant seeks
causal relationship between him and Rodel or relief based on a quasi-delict under
the vehicle used that will make him Articles 2176 and 2180 of the Civil
accountable for Marvins death. Mayor Favis Code.
was a mere passenger at the time of the
accident. It is the option of the plaintiff whether
to waive completely the filing of the civil
Parenthetically, it has been held that action, or institute it with the criminal action,
the failure of a passenger to assist the driver or file it separately or independently of a
by providing him warnings or by serving as criminal action; his only limitation is that he
lookout does not make the passenger liable for cannot recover damages twice for the same
the latters negligent acts. The drivers duty is act or omission of the defendant. (PCI Leasing
not that may be delegated to others. and Finance, Inc v. UCPB General Insurance Co.
Inc., G.R. No. 162267, July 4, 2008)
85. Who is liable for Marvins death?
87. D owes C P100, 000. Upon maturity of
Liability attaches to the registered the loan, D fails to pay and so C sues him in a
owner (Virgilio), the negligent driver (Rodel) complaint for sum of money. D answers the
and the latters employer (Municipality of complaint and before actual hearing, C
Magsingal). Settled is the rule that the assigns the promissory note signed by D to E
registered owner of a vehicle is jointly and for P80, 000. The assignee, E, now demands
severally liable with the driver for damages payment from D. For how much is D obliged
incurred by passengers and third persons as a to pay E?
consequence of injuries or death sustained in
the operation of said vehicles. Regardless of

BAR OPERATIONS 2013


I press toward the mark for the
prize of the high calling of God in
Christ Jesus. - Philippians 3:14

BARRISTERS CLUB Page 25 of 55


in CIVIL LAW

Prepared by: Atty. Roney Jone P. Gandeza

D is liable only for P80, 000, plus cost


and interest. Under Article 1634 of the Civil Yes, A can be considered a builder in
Code, when a credit in litigation is sold, the good faith even if his lot is covered by a
debtor shall have a right to extinguish it by Torrens Title. Good faith is a state of mind.
reimbursing the assignee for the price the Unless one is versed in the science of
latter paid therefor, the judicial costs incurred surveying, no one can determine the precise
by him, and the interest on the price from the extent or location of his property by merely
day on which the same was paid. In paying examining his paper title.
only such amount, D exercises his right of legal
redemption. 90. Is C, As buyer, entitled to avail of the
benefits under Article 448?
(RJPG: The right of redemption
granted in Article 1634 of the Civil Code is Yes. Upon delivery of the property to
proper only in case of sale [of the credit in him, C acquired ownership of the property and
litigation], and not to cases of barter, he is deemed to have stepped into the shoes
donation, or other modes of acquisition.) of the seller, A, in regard to all the rights of
ownership, including the right to compel B, the
88. A purchased a property adjacent to owner of the lot encroached upon, to exercise
that of B. Shortly thereafter, A discovered either of the two options granted to him by
after relocation survey that a 30-square meter law after payment of proper indemnity or to
portion of his lot sis occupied by Bs house. sell the portion encroached upon.

Despite repeated demands, B refused 91. Is B entitled to demand the removal


to vacate the encroached portion. He claims of the encroaching structure?
that under Article 448 of the Civil Code, he
has the pre-emptive right to purchase the B, the lot owner, cannot demand the
portion encroached upon. removal of the encroaching structure. Such
right is available only if and he chooses to
Is B correct? compel the builder to buy the land at a
reasonable price but the latter fails to pay it.
No, he is not. Article 448 of the Civil (Technogas Phil. V. Court of Appeals, 268 SCRA
Code is unequivocal that the option to sell the 5 [1997])
land on which another builds, plants or sows in
good faith, belongs to the landowner. This (RJPG: If B decide to appropriate the
advantage in Article 448 of the Civil Code is improvements, the builder has the right to
accorded the landowner because his right is retain the lot until he is paid of his necessary
older, and by the principle of accession, he is and useful expenses. He is not even required
entitled to the ownership of the accessory to pay rentals in the meantime.)
thing. (Benitez v. C, 226 SCRA 242)
92. Aragon is indebted to Benitez and
89. A owns a parcel of registered land Chua in the amount of P200, 000. Upon
adjacent to that of B. A builds a house on his maturity of the debt, Aragon fails to pay and
lot. Unknown to A, however, a portion of his so Benitez and Chua sue him in a complaint
house has encroached on Bs property. After for sum of money. Aragon answers the
the construction, A sold his lot to C. complaint and before actual hearing, Benitez
assigns his right to the credit to Chua
Is A a builder in good faith under (presumably or P100, 000) for only P75,
Article 448 of the Civil Code?

BAR OPERATIONS 2013


I press toward the mark for the
prize of the high calling of God in
Christ Jesus. - Philippians 3:14

BARRISTERS CLUB Page 26 of 55


in CIVIL LAW

Prepared by: Atty. Roney Jone P. Gandeza

000. For how much is Aragon obliged to pay support pendent lite. Peter at that time has
Chua? settled in the Philippines with his new wife
Maria, a Filipina.
Aragon is liable to pay P200, 000 to
Chua because the assignment was made to a Will the case prosper?
co-owner. In other words, Aragon cannot
redeem the credit in litigation sold by Benitez The case will not prosper. In Bayot v.
to Chua. Article 1635 of the Civil Code Court of Appeals, G.R. No. 155635, November
enumerates the three instances when the 7, 2008, the Supreme Court said that the
debtor cannot redeem a credit in litigation divorce decree secured by Peter in Nevada
which is sold by his creditor, one of them being shall be given a res judicata effect in this
an assignment or sale to a co-owner. jurisdiction. As an obvious result of the divorce
decree obtained, the marital vinculum
93. XYZ Insurance Company insured between Peter and Sonia is considered
Pedros house for Php 500, 000. In the policy, severed; they are both free from the bond of
the insurer undertakes, upon total loss, to matrimony. In plain language, Peter and Sonia
either pay the insure value of the house, or are no longer husband and wife to each other.
rebuild it, upon proof of total loss. If during Consequent to the dissolution of the marriage,
the life of the policy the insure property is Peter could no longer be subject to a
completely destroyed, may the insured insist husbands obligation under the Family Code.
the insurance company rebuild his house than He cannot, for instance, be obliged to live with,
being paid its insured value? observe respect and fidelity, and render
support to Sonia. With the valid foreign
No, because in alternative obligations, divorce secured by Peter, there is no marital
the right of choice is given to the debtor, tie binding him to Sonia. There is on fine no
unless it has been expressly granted to the more marriage to be dissolved or nullified.
creditor (par. 1 Art 1200, Civil Code). In the
absence of an agreement in the insurance 95. H designated his wife as his universal
policy giving the right of choice to the insured, heir (no other compulsory heirs existed) on
the general rule applies, and therefore, the condition that when she became a widow,
insurer may choose which of the two she must never remarry. Two years after H
prestations to perform, the performance of died, the widow remarried. Is she entitled to
one being sufficient. the inheritance?

94. In 1994, Sonia, a Filipina, went to The condition is valid insofar as the
work as a nurse in New York. There she met free portion is concerned, since this absolute
and fell in love with Peter, an American, prohibition was imposed by a deceased spouse
whom she married in 1996. Sonia acquired but is not valid insofar as her legitimate is
American citizenship in 1998. Due to concerned. Therefore, her remarriage makes
irreconcilable differences, however, the her lose the free portion, but not the
couple parted ways in 2000. The following legitimate (for ordinarily, no condition can be
year, Peter obtained a valid divorce decree in imposed on the legitime. (Art. 874, CC)
Nevada. Heartbroken, Sonia repatriated to
the Philippines in 2001 and reacquired Filipino 96. H instituted his widowed sister as his
citizenship that same year. Sonia heir on condition that the latter will not
subsequently filed an action against Peter for marry again. When H died, his sister
declaration of nullity of marriage under remarried. Is she entitled to the inheritance?
Article 36 of the Family Code with prayer for

BAR OPERATIONS 2013


I press toward the mark for the
prize of the high calling of God in
Christ Jesus. - Philippians 3:14

BARRISTERS CLUB Page 27 of 55


in CIVIL LAW

Prepared by: Atty. Roney Jone P. Gandeza

Yes, because the condition is deemed validity of said marriage, so long as it is


not written. The rule under Article 874 of the essential to the determination of the case.
Civil Code is that an absolute condition not to However, evidence must be adduced,
contract a first or subsequent marriage shall be testimonial or documentary, to prove the
considered as not written unless such existence of grounds rendering such a
condition has been imposed on the widow or marriage an absolute nullity.
widower by the deceased spouse, or by the
latters ascendants or descendants. The In the instant case, it is clear that
condition that the sister shall not remarry for Zandro and Jessica did not have a marriage
her to get the inheritance is void because it is license when they contracted their marriage.
contrary to good morality and public policy. Instead, they presented a false affidavit stating
that they had been living together for more
QUESTIONS 97-98 are based on the than five years. For this reason, their marriage
following fact situation: is deemed void ab initio.

Jessica institutes an action against 98. What is the legal standing of the false
Zandro for support in the allegation that they affidavit executed by Zandro and Jessica?
are husband and wife and that Zandro has
reneged on his obligation to financially The falsity of the affidavit cannot be
sipport her as his wife and Tricia as their considered as a mere irregularity in a formal
child. Zandro denies that he is married to requisite of marriage. The law dispenses with
Jessica. He avers that their marriage is void the marriage license requirement for a man
because it was solemnized without a marriage and woman who have lived together and
license; that the marriage ceremony was exclusively with each other as husband and
facilitated by an affidavit wherein they falsely wife for a continuous and unbroken period of
stated that they had been living together as at least five years before the marriage. The
husband and wife for at least five years; and false affidavit which Zandro and Jessica
that they have never cohabited as husband executed so they could push through with the
and wife. marriage has no value whatsoever; it is a mere
scrap of paper. They were not exempt from
97. May a party to an action collaterally the marriage license requirement. Their failure
attack the validity of a marriage as what to obtain a present marriage license renders
Zandro had asserted in his answer? their marriage void ab initio. (De Castro v. De
Castro, GR No. 160172, February 13, 2008)
Yes. The court has jurisdiction to
determine the validity of the marriage of 99. X obtains a loan from Y. They agree
Zandro and Jessica. More appropriately, the that upon maturity of the loan, X will give
validity of their void marriage may be either the sum owed or a particular house
collaterally attacked. Thus, in Nicdao Carino v. and lot. X now assails the agreement as a
Yee Carino, 403 SCRA Phil. 861 [2001], the pactum commissorium. Is X correct?
Supreme Court declared that courts are
clothed with sufficient authority to pass upon The stipulation is simply an alternative
the validity of two marriages despite the main obligation, which is expressly allowed by the
case being a claim for death benefits. law. The agreement to convey the house and
Reiterating Ninal v. Badayog, 384 Phil. 661 lot in the event of Xs failure to pay the debt in
[2000], the Supreme Court held that the courts money at its maturity does not constitute
may pass upon the validity of marriage even in pactum commissorium.
a suit not directly instituted to question the

BAR OPERATIONS 2013


I press toward the mark for the
prize of the high calling of God in
Christ Jesus. - Philippians 3:14

BARRISTERS CLUB Page 28 of 55


in CIVIL LAW

Prepared by: Atty. Roney Jone P. Gandeza

It is NOT an attempt to permit the the ceiling joists to admit light in 1995. Even
creditor, Y, to declare forfeiture of ther after 10 years, B may still obstruct the light by
security upon the failure of the debtor to pay constructing on his own lot a building higher
the debt at maturity. It simply provided that if than As unless A makes a notarial prohibition
the debt is not paid in money, it shall be paid prohibiting B from making the construction.
another way. Of in 23001 A makes the prohibition, may still
B make the obstruction in 2007?
100. D borrowed from C P5, 000 payable in
one year. When C was abroad, Cs 16-year old Yes, because it is only in 2013 (ten
son borrowed P2, 500 from D for his school years after the notarial prohibition) when A
tuition. However, the son spent the money on may be said to have acquired the negative
a cellular phone. When the debt to C fell due, easement of light and view. After 2013, B may
D tendered only P2, 500 claiming no longer obstruct.
compensation on the P2, 500 borrowed by Cs
son. Is D legally entitled to claim partial legal 103. About 15 years ago, A constructed a
compensation? house on his lot adjoining the lot owned by B.
He provided it with windows overlooking Bs
No. This is so because under Articles lot half a meter away from the boundary line.
1278 and 1279 of the Civil Code, in order that A month ago, B brought an action against A
there will be a valid and effective for the closure of the windows on the
compensation, it is essential that there must allegation that they violate the law on
be two parties who in their own right are distances.
principal creditors and principal debtors of
each other. Has A acquired an easement of light
of view by prescription of ten years?
In the instant case, C cannot be
considered as a party to the act of his son in No. In the first place, there was no
borrowing P2, 500 from D. Consequently, he formal prohibition under which A prohibited B
did not become a principal debtor of D; neither from obstructing his light and view.
did D become a principal creditor of C.
Therefore, there can be no partial In the second place, A did not observe
compensation of the P5, 000 borrowed by D the legal requirement that there should be a
from C. distance of at least two meters between the
windows and Bs lot, since the view is direct.
101. Would the answer be the same if Cs According to the Civil Code, non-observance of
son actually used the money for his school this distance does not give rise to prescription.
tuition?
104. If the vendee in a contract of sale
There would be no difference in the expressly renounces the right to warranty in
answer. The fact that Cs son actually used the case of eviction, and his eviction should take
P2, 500 for his school tuition did not make C a place, can he still hold the vendor liable?
party to the contract between his son and D.
Therefore, C is not the principal debtor of D It depends. In case or renunciation of
and D is not the principal creditor of C with the warranty without knowledge of the risks of
respect to the subject amount. eviction, the vendor is only bound to pay the
value of the thing at the time of the eviction.
102. A and B are neighbors. On his Although as a consequence of the waiver, the
buildings wall, A opened a window beneath vendor is not bound to indemnify the vendee

BAR OPERATIONS 2013


I press toward the mark for the
prize of the high calling of God in
Christ Jesus. - Philippians 3:14

BARRISTERS CLUB Page 29 of 55


in CIVIL LAW

Prepared by: Atty. Roney Jone P. Gandeza

fully in accordance with Article 1555 of the words, the basis of Abes liability is his implied
Civil Code still the effect of the waiver cannot contract with the painters.
be extended as to exempt the vendor from
returning the price. When eviction occurs, the (RJPG: It is error to state that Abe is
contract is left without cause as to the vendee liable because of the benefit bestowed; or that
and inasmuch as his obligation to pay the price his liability is grounded on the existence of a
is condition upon the delivery of the thing by quasi-contract; or that he is obliged to pay the
the vendor, from the moment the vendee is painters to prevent unjust enrichment. This is
deprived of the possession of the thing, the so because of the implied contract between
payment of the price really becomes a him [Abe] and the painters.)
payment of what is not due which, under
Article 2154 of the Civil Code, should be 107. Suppose Abe had not been at home,
returned. and he did not recover that the fence had
been painted until he returned in the evening,
105. A appoints B to sell his registered is Abe liable? If so, what would be the basis of
land. B negotiates the sale with C. Unknown his liability?
to B, A also negotiates the sale with D,
thereby making contracts incompatible with Abe would not be obliged in this
each other. Who is now the owner of the situation. HE did not learn of the benefits
land, C or D? bestowed until after they had been completed.
There had been no communication of the offer
Article 1916 of the Civil Code of the painters and no express or implied
establishes the rule of preference when two acceptance by Abe of the work done. Neither
persons contract with regard to the same is Abe liable to the painters under quasi-
thing, one of them with the agent and the contract. This is so because there was neither a
other with the principal. Under this provision, case of colution indebiti or negotiorum gestio
that of prior date shall be preferred, without in the instant case.
prejudice to the provisions of Article 1544 of
the Civil Code. Consequently, whoever 108. Apolinario Mabini died intestate in
between C and D registers the sale on good 1995, leaving his wife, Dorothy, four
faith is the owner of the land; and in the legitimate children, and considerable
absence of registration, the first to take properties which they divided among
possession of the land in good faith. themselves. Claiming to be an illegitimate son
of the deceases Apolinario, and having been
106. While sitting on his front porch, Abe left out in the extrajudicial settlement of
watched three men paint a fence on his Apolinarios estate, Mario instituted an action
property. The men later learned that they had for partition against Dorothy and her children.
made a mistake. The fence they were to have
painted was in the next block, in the property At the trial, Mario admitted that he
of Rey. Is Abe obliged to pay the painters? had none of the authentic documents
mentioned in Article 172 of the Family Code
By his toleration of the trespassers and to show that he was the illegitimate child of
by his failure to protest, Abe is obliged to pay Apolinario. Is this admission sufficient basis
the painters the reasonable value of the work for Dorothy and her children to move for the
done and the materials used. The basis of his dismissal of Marios complaint?
liability would be his implied acceptance of the
offer of the painters to paint his fence. In other Yes. An illegitimate child, like Mario, is
allowed by law to establish his illegitimate

BAR OPERATIONS 2013


I press toward the mark for the
prize of the high calling of God in
Christ Jesus. - Philippians 3:14

BARRISTERS CLUB Page 30 of 55


in CIVIL LAW

Prepared by: Atty. Roney Jone P. Gandeza

filiation either by an authentic document or by


any other means allowed by the Rules of 110. Abe is a bell boy at the Pines View
Court and special laws, like his baptismal Hotel. At the point of a gun, he entered Reys
certificate, a judicial admission, a family Bible hotel room and robbed Rey and his wife of
in which his name has been entered, common their valuables. The hotel disclaimed liability
reputation respecting his pedigree, admission on the ground of force majeure. Is the hotel
by silence, testimonies of witnesses, and other liable?
kinds of proof admissible under Rule 130 of the
Rules of Court. Yes. It is true that here the robbery
was committed with the use of arms, (which
The problem of Mario, however, is would have been a case of force majeure) but
that, since he seeks to prove his filiation under then Abe was an employee of the hotel. The
the second paragraph of Article 172 of the hotel will be liable not because of force
Family Code, his action is now barred because majeure under Article 2001 which evidently
of his alleged fathers death in 1995. This refers to a stranger, but because of Article
particular provision of law specifically provides 2000 of the Civil Code which provides that the
that when the action to claim illegitimate master is responsible for the acts of his
filiation is based on the second paragraph of servants.
Article 172, the action must be brought during
the lifetime of the alleged parent. 111. Arnel pledges his Nokia cellular phone
to Bert to secure a loan. Arnel and Bert agree
It is clear, therefore, that Mario can no that the latter could purchase the cellular
longer be allowed at this time to introduce phone at the current purchase price if the
evidence of his open and continuous debt is not paid on time. Arnel assails the
possession of the status of an illegitimate child agreement for being a pactum commissorium.
or prove his alleged filiation through any of the Is Arnel correct?
means allowed by the Rules of Court or special
laws. The simple reason is that Apolinario is No. The stipulation is not a pactum
already dead and can no longer be heard on commissorium. What is prohibited by Article
the claim of his allged sons illegitimate 2008 of the Civil Code, dealing with pactum
filiation. (Uvguanco v. Court of Appeals. G.R. commissorium, is the automatic appropriation
No. 76873, Ocotber 26, 1989) by the creditor or pledge of the thing pledged
in payment of the loan at the expiration of the
109. Rey is a registered guest at the Pines period agreed upon. Where there is an express
View Hotel in Baguio City. In the middle of the authorization on the creditor to purchase the
night, Abe, a hotel guest himself, went up the thing pledged at the current market price, the
fire escape, slowly raised the window in Reys contract would not come within the
room, went inside the room, and stole Reys prohibition as there is no automatic
expensive laptop. The hotel disclaimed appropriation by the creditor of pledge of the
liability on the ground of force majeure. Is the thing pledged.
hotel liable?
112. Is there an instance under the law
Yes. The problem involves a case of where the creditor is allowed to appropriate
robbery with force upon things. Under Article for himself the thing given by way of security
2001 of the Civil Code, the act of a thief or because of non-payment of the debt?
robber, who has entered the hotel is not
deemed force majeure, unless it is done with Yes, in the case of pledge. If at the first
the use of arms or through an irresistible force. auction the thing pledged is not sold, a second

BAR OPERATIONS 2013


I press toward the mark for the
prize of the high calling of God in
Christ Jesus. - Philippians 3:14

BARRISTERS CLUB Page 31 of 55


in CIVIL LAW

Prepared by: Atty. Roney Jone P. Gandeza

auction must be held. If the thing pledged is infant whose parents were unknown was
not sold at second auction, the creditor is now entrusted to them by a friend. Eager to have a
allowed to appropriate for himself the thing child of their own, Rey and Mimi registered
pledged but he must give an acquittance for the child to make it appear that they were the
his entire claim. (Art. 2112, Civil Code) childs parents. They named the infant Angela
and followed Reys surname.
113. When the proceeds of the sale of the
mortgaged property in chattel mortgage does The spouses reared and cared for
not fully satisfy the debt, is the mortgage Angela as if she was their own. They sent the
entitled to recover the deficiency from the child to exclusive schools, and used the
mortgagor? surname of Rey on all her school records and
documents. Rey died in 1998. In 2000, Mimi
Yes. It is a settled rule that if the married Abe, an American citizen. Shortly
proceeds of the sale are insufficient to cover thereafter, Mimi decided to adopt Angela by
debt either in an extrajudicial or judicial availing of the amnesty given under the law
foreclosure of mortgage, the mortgagee is to those individuals who simulated the birth
entitled to claim deficiency from the debtor. of a child. Thus, in 2002, Mimi filed a petition
While the legislature has denied the right of a for the adoption of Angela. Abe executed an
creditor to sue for deficiency resulting from affidavit giving his consent to the adoption of
foreclosure of security given to guarantee an Angela.
obligation as on the case of pledges (Art. 2115,
Civil Code) and in chattel mortgages of a thing 115. Can Mimi alone adopt Angela?
sold on installment basis (Art. 1484, par. 3,
Civil Code), and the law does not prohibit Mimi alone cannot adopt Angela. At
recovery of deficiency. Accordingly, a the time the petition for adoption was filed,
deficiency claim arising from the extrajudicial Mimi had already remarried. She filed the
foreclosure of mortgage is allowed. (PNB v. petition by herself, without being joined by her
Court of Appeals, 308 SCRA 229 [1999]) husband, Abe. The law is explicit. Section 7,
Article III of RA No. 8552 specifically provides
114. May a legitimate child impugn his that the husband and wife shall jointly adopt,
legitimate status? except if one spouse seeks to adopt the
legitimate child of the other; or if one spouse
No, the law itself establishes the seeks to adopt his own illegitimate child; or if
legitimacy of a child conceived or born during the spouses are legally separated from each
the marriage of his parents. The presumption other.
of legitimacy fixes a civil status for the child
born in wedlock, and only the father (Art. 160, Mimi does not fall under any of the
Family Code), or in exceptional instances the three exceptions enumerated in Section 7.
latters heirs (Art. 171, Family Code), can First, the child adopted is not the legitimate
contest in an appropriate action the legitimacy child of Mimi or of her husband Abe. Second,
of a child. A child cannot choose his own the child is not the illegitimate child of Mimi.
filiation. And third, Mimi and Abe are not legally
separated from each other.
QUESTIONS 115-116 are based on the
following fact situation: 116. Suppose Abe filed for divorce during
the adoption proceedings, would that make a
Rey and Mimi were married in 1985. difference in you answer?
They were childless. By stroke of fate, an

BAR OPERATIONS 2013


I press toward the mark for the
prize of the high calling of God in
Christ Jesus. - Philippians 3:14

BARRISTERS CLUB Page 32 of 55


in CIVIL LAW

Prepared by: Atty. Roney Jone P. Gandeza

It will not make any difference in my


answer. The filing of a case for dissolution of 118. What is the doctrine of dependent
the marriage between Mimi and Abe is of no relative revocation in the law on succession?
moment. It is not equivalent to a decree of
dissolution of marriage. Until and unless there If the testator revokes his will with the
is a judicial decree for the dissolution of the present intention of making a new one and the
marriage between Mimi and Abe, the marriage new will is not made, or if made, fails to effect
still subsists. That being the case, joint for any reason whatsoever, it will be presumed
adoption by the husband and the wife is that the testator prefers the old will to
required. To repeat; since at the time the intestacy. The old will can still be admitted to
petition for adoption was filed, Mimi was prbate.
married to Abe, joint adoption is mandatory.
(In re: Petition for adoption of Michele P. Lim Stated otherwise, the revocation is
GR Nos. 168992-93 May 21, 2009) subject to a SUSPENSIVE CONDITION: That the
testator will make a new will and that such will
117. Venus secures a judgment against shall take effect. If such condition is not
Erlinda for the payment of civil liability arising fulfilled, then there is no revocation.
from the crime of slander committed by
Erlinda against Venus. To satisfy the 119. The Domestic Adoption Act (RA 8552)
judgment, the sheriff levies on a real property requires, inter alia, that before an alien can
owned by the conjugal partnership of Erlinda adopt in the Philippines, he must have
and her husband, Romulo. May the property resided in the Philippines for at least three
be sold on execution? continuous years prior to the filing of the
application for adoption and maintains such
The property is exempt from residence until the adoption decree is
execution. Article 122 of the Family Code entered. Are there exceptions to this
explicitly provides that payment of personal requirement?
debts contracted by the husband or the wife
before or during the marriage shall be charged Yes, they are as follows:
to the conjugal partnership, except insofar as
they redounded to the benefit of the family. By (a) The alien is a former Filipino citizen
no stretch of imagination can it be concluded who seeks to adopt a relative within the fourth
that the civil obligation arising from the crime civil degree of consanguinity;
of slander committed by Erlinda redounded to
the benefit of the conjugal partnership. (b) The alien seeks to adopt the
legitimate or illegitimate child of his or her
Unlike in the system of absolute Filipino spouse; and
community where liabilities incurred by either
spouse by reason of a crime or quasi-delict is (c) The alien is married to a Filipino
chargeable to the absolute community of the citizen and seeks to adopt jointly with his or
property, in the absence or insufficiency of the her spouse a relative within the fourth civil
exclusive property of the debtor-spouse, the degree of consanguinity or affinity of the
same advantage is not accorded in the system Filipino spouse.
of conjugal partnership of gains. The conjugal
partnership of gains has no duty to make 120. Abe is the son of Fidel with his first
advance payments for the liability of the wife, while Rey is Fidels son with his second
debtor-spouse. (Sps. Buado v. Court of wife. Both wives predeceased Fidel. Upon
Appeals, 145222, April 24, 2009) Fidels death in 2008, Abe immediately

BAR OPERATIONS 2013


I press toward the mark for the
prize of the high calling of God in
Christ Jesus. - Philippians 3:14

BARRISTERS CLUB Page 33 of 55


in CIVIL LAW

Prepared by: Atty. Roney Jone P. Gandeza

instituted and action for partition of Fidels (Arriola et. al. v Arriola, GR No. 177703,
estate. After trial in due course, the court January 28, 2008)
rendered judgment ordering the partition in
equal shares between Abe and Rey the land (RJPG: The Supreme Court declared in
exclusively owned by Fidel. Honrado v. Court of Appeals, 476 SCRA 280
[2005], that a claim for exception from
As Abe and Rey failed to agree on execution or forced sale under Article 153
how to partition the property, the court should be set up and proved before the sale of
ordered its sale at public auction. However, the property at public auction. In the above-
the public sale did not push through because entitled case of Arriola, Dos timely objected to
Rey refused to include in the auction sale the the inclusion of the subject house although for
house standing on the land on the allegation a different reason.
that the house has been residence nearly 20
years, and has thus acquired the status of a 121. Distinguish between right of
family home. Rey also pointed out that since redemption and equity of redemption.
the house was not mentioned in Abes
complaint for judicial partition, such house is Equity of redemption is the right of the
not susceptible of partition. Decide. mortgagor after judgment in a judicial
foreclosure to redeem the property by paying
The house is deemed included in the to the court the amount of the judgment debt
judgment of partition, and this is true even if before the sale or confirmation of the sale. On
its existence was not mentioned in Abes the other hand, right of redemption is the right
complaint. Pursuant to law, since Fidel owned of the mortgagor to redeem the property sold
the land, he also owned the house which is a at an extrajudicial foreclosure sale by paying to
mere accessory to the land. Both properties the buyer in the foreclosure sale the amount
form part of the estate of the deceased and paid by the buyer within one year from such
are held in co-ownership by his heirs. Any sale.
decision in the action for partition of said
estate would cover not just the subject land 122. May the true owner of a movable
but also the subject house. property recover possession of his property
from the present possessor? If so, is there a
While it is conceded that the subject need to reimburse said possessor?
house is covered by the judgment of partition,
the ruling does not necessarily countenance The true owner of a movable property
the immediate and actual partition of said may recover possession of his property
property by way of public auction in the view without reimbursement from a possessor in
of the suspensive prescription imposed under bad faith or even from possessor in good faith
Article 159 of the Family Code. if said owner had lost the property or been
unlawfully deprived of it, the acquisition being
Set against the foregoing rule, the from a private person.
family home consisting of the subject house
and lot on which it stands- cannot be The owner may also recover
partitioned at this time, even if it has passed to possession of his movable property but should
the co-ownership of his heirs. Fidel died in reimburse the possessor if such possessor
2008. Thus, for 10 years from said date or until acquired the property in good faith at a public
2018, or for a longer period, if there is still a sale or auction (Art. 559, CC)
minor beneficiary residing therein, the family
home he constituted cannot be partitioned.

BAR OPERATIONS 2013


I press toward the mark for the
prize of the high calling of God in
Christ Jesus. - Philippians 3:14

BARRISTERS CLUB Page 34 of 55


in CIVIL LAW

Prepared by: Atty. Roney Jone P. Gandeza

However, the owner can no longer announcement of the finding for two
recover possession of his movable property, consecutive weeks. Six months from the
even if he offers to reimburse, whether or not publication having elapsed without the owner
he had lost his property or had been unlawfully having appeared, the thing found shall be
deprived of it, if the possessor had acquired awarded to A, after reimbursement of the
the property in good faith by purchase from a expenses. (Art. 719, CC) Should the owner
merchants store, or in fairs, or markets in appear in time, he shall be obliged to pay A, as
accordance with the Code of Commerce and a reward, 1/10 of the amount found. (Art. 720,
Special Laws (Art. 1505, Civil Code and Arts. 85, CC)
86 Code of Commerce); or if the owner is
precluded from denying the sellers authority, 125. When can the owner of an estate
or if the possessor had obtained the goods claim a compulsory right of way?
because he was an innocent purchaser for Under Articles 649 and 650 of the Civil
value and a holder of a negotiable document Code, the owner of an estate may claim a
title to the goods. (Art. 1518, CC) compulsory easement of right of way after he
has established the existence of the following
123. Why is a negative easement requisites: (a) the estate is surrounded by
acquirable by prescription despite its non- other immovables and is without an adequate
apparent? outlet to a public highway; (b) proper
indemnity is paid; (c) the isolation is not due to
Generally, negative easements cannot the proprietors own acts; and (d) the right of
be acquired by prescription because they are way claimed is at a point least prejudicial to
non-apparent. Still, the very existence of the the servient estate and insofar as consistent
Civil Code (insofar as it relates to negative with the law, where the distance from the
easements), prove that in certain cases, and dominant estate to a public highway may be
for purposes of prescription, there are the shortest.
negative easements that may be indeed be
considered apparent, not because there are 126. May an existing easement of right of
visible signs of existence but because of the way be extinguished by the opening of an
making a notarial prohibition. The notarial adequate outlet to a public highway?
prohibition makes apparent what really is non-
apparent. An easement of right of way provided
for in a contract of sale is a voluntary
124. While walking along Session Road in easement. As such, it cannot be extinguished
Baguio City, A found a purse containing P50, by the opening of an adequate outlet to a
000. Can A become the owner of the P50, public highway. The opening of an adequate
000? outlet can extinguish a legal or compulsory
easement but not a voluntary easement. (La
If A knows the owner of the purse, Vista Association v. Court of Appeals, 278 SCRA
there is no way by which he can become the 498 [2000])
owner of the P50, 000 because under the law,
he must return the purse, including its 127. B donated to C a parcel of land
contents, to the owner. subject to a condition. When C failed to
comply with the condition, B sold the land to
If the owner is unknown, A shall D. Is the sale an act of revocation of the
immediately deposit the movable with the donation?
mayor of the place where the purse was
found. There shall then be a public

BAR OPERATIONS 2013


I press toward the mark for the
prize of the high calling of God in
Christ Jesus. - Philippians 3:14

BARRISTERS CLUB Page 35 of 55


in CIVIL LAW

Prepared by: Atty. Roney Jone P. Gandeza

No. The act of selling the property mutuality. After all, the lessor is free to give or
donated cannot be considered as a valid act of not to give the option to the lessee. And while
revocation of the donation because a formal the lessee has a right to elect whether to
action in court to revoke the donation must be continue with the lease or not, once he
filed by the donor pursuant to Article 764 of exercises his option to continue and the lessor
the Civil Code which speaks of an action that accepts, both parties are thereafter bound by
has a prescriptive period of four (4) years from new lease agreement. Their rights and
non-compliance with the condition stated in obligations become mutually fixed, and the
the donation. The rule that there can be lessee is entitled to retain the possession of
automatic revocation without the benefit of a the property for the duration of the new lease,
court action does not apply in this case and the lessor may hold him liable for the rent
because the subject donation is devoid of any thereof. Mutuality obtains in such a contract
provision providing for automatic revocation in and equality between the lessor and the lessee
the event of con-compliance with the since they remain with the same faculties in
condition set forth therein. Thus, a court respect to fulfillment.
action is necessary to be filed within four (4)
years from the non-compliance with the 130. How shall the clause may be
condition violated. renewed for a like term at the option of the
lessee be interpreted or applied?
128. May the heirs of the donor sue the
heirs of the done for revocation of the The clause means that the exercise by
donation if there is a violation of any the lessee of his option resulted in the
restriction in the deed of donation? automatic extension of the contract of lease
under the same terms and conditions
Yes. Under Article 764 of the Civil prevailing in the original contract of lease, i.e.,
Code, the donor or his heirs have the for 14 years, the phrase for a like term
personality to question the violation of any referring to the term of the lease. If the
restriction in the deed of donation. renewed contract were still subject to mutual
Consequently, the right to revoke be agreement by the lessor and the lessee, then
transmitted to the heirs of the donor and may the option which is an integral part of the
be exercised against the heirs if the done, and consideration for the contract would be
the action prescribes after four years from the rendered worthless. For then, the lessor could
violation of the condition. easily defeat the lessees right by simply
imposing unreasonable and onerous
129. A lease agreement contains the conditions to prevent the parties from
following stipulation: That the term of the reaching an agreement. (Allied Banking Corp.
lease shall be 14 years beginning April 1, 1998 v. Court of Appeals, 284 SCRA 357 [1998])
and may be renewed for a like term at the
option of the lessee. The lessor contends 131. May a lessee invoke the application of
that this stipulation is void because it is Article 16887 of the Civil Code for the
violative if the principle of mutuality of extension of his lease in the same ejectment
contracts under Article 1308 of the Civil Code. case filed by his lessor?
Is the lessor correct?
Yes. The exercise of the power given to
No. The fact that the lessees option to the courts in Article 1687 of the Civil Code to
renew the lease is binding only on the lessor extend the period of the lease when the lessee
and can be exercised only by the lessee does has been in occupancy of the premises for
not render such option void for lack of more than a year does not contemplate a

BAR OPERATIONS 2013


I press toward the mark for the
prize of the high calling of God in
Christ Jesus. - Philippians 3:14

BARRISTERS CLUB Page 36 of 55


in CIVIL LAW

Prepared by: Atty. Roney Jone P. Gandeza

separate action for that purpose. That power is deemed terminated. This kind of contractual
may be exercised as an incident in the action stipulation is not illegal, there being nothing in
for ejectment itself and by the court having the law proscribing such kind of agreement.
jurisdiction over it.
Moreover, judicial permission to
132. May a common carrier be held liable cancel the lease agreement is not, therefore,
for the death or injury of a passenger caused necessary because of the express stipulation in
by the willful acts or negligence of another the contract of lease that the lessor, in case of
passenger or stranger? failure of the lessee to comply with the terms
and conditions thereof, can take over the
Yes, a common carrier is responsible possession of the leased premises, thereby
for the death or injury of a passenger caused cancelling the contract of lease. Resort to
by the willful act or negligence of another judicial action is necessary only in the absence
passenger or stranger, but only if the of a special provision granting the power of
employees of the common carrier through the cancellation.
exercise of diligence of a good father of family
could have prevented the act or omission. 135. D borrowed from C P350, 000.00
payable in three months. To secure payment
133. When does an action for of the loan, D executed in Cs favor a Real
reconveyance based on implied or Estate Mortgage. From the loan amount that
constructive trust prescribe? D was to receive, P17, 500.00 was pre-
deducted as interest for the first month which
An action for reconveyance of a parcel was equivalent to 5% of the principal debt,
of land based on an implied or constructive and P7, 500.00 equivalent to 1.79% of the
trust prescribes in ten years, the point of debt, was further deducted as service fee.
reference of being the date of registration of Thus, D actually received only P325, 000.
the deed or the date of issuance of the
certificate of title over the property. However, When D failed to pay the loan, the
this rule applies only when the plaintiff or the loan was extended through a restructuring
person enforcing the trust is not in possession agreement in such a way that the unpaid
of the property, since if a person claiming to be interest became part of the principal. The
the owner thereof is in actual possession of restructuring agreement adopted all other
the property, the right to seek reconvenyance terms and conditions contained in the first
which in effect seeks to quiet title to property, loan agreement.
does not prescribe. Due to the continued inability of D to
pay, the loan agreement was renewed to five
134. A contract of lease contains a more times. In all these subsequent renewals,
stipulation authorizing the lessor to take over the same terms and conditions found in the
possession of the leased premises without first agreement were retained. Consequently,
judicial intervention upon failure of the lessee the principal was finally increased to P880,
to pay rent. Is the stipulation valid? 000. This increase in the amount of principal
had been due to unpaid interest and other
The validity of a contractually- charges.
stipulated termination clause has been upheld
by the Supreme Court. The stipulation is in the When the debt remained unpaid, C
nature of resolutory condition, for upon the formally demanded from D the payment of
exercise by the lessor of his right to take P880, 000. When the demand was ignored, C
possession of the leased property, the contract foreclosed the mortgage. At the auction sale,

BAR OPERATIONS 2013


I press toward the mark for the
prize of the high calling of God in
Christ Jesus. - Philippians 3:14

BARRISTERS CLUB Page 37 of 55


in CIVIL LAW

Prepared by: Atty. Roney Jone P. Gandeza

the mortgaged property was sold to C, the reconveyance. (Espiritu v. landrito, GR Nos.
lone bidder, for the amount of P880, 000. 169617, April 4, 2007)

Is the foreclosure sale valid? What is 136. Donor donates on January 1. Donee
Ds remedy in law? accepts on January 5. Donor dies on January
8. Acceptance of donation reached donors
The foreclosure is void. Since D was house on January 10. Is there a perfected
not given an opportunity to settle his debt, at donation?
the correct amount and without the iniquitous
interest imposed, no foreclosure proceedings There is no perfect donation because done
may be instituted. A judgment ordering a died at the time of the making of the
foreclosure sale is conditioned upon a finding donation.
on the correct amount of the unpaid obligation
and the failure of the debtor to pay the said 137. Insane donor donates on January 1.
amount. (Secs. 2 and 3, Rule 68, Rules of Court) Donee accepts on January 5. Donor becomes
san on January 7. Donor receives acceptance
In this case, it has not yet been shown on January 8. Is there a perfect donation?
that D had already failed to pay the correct
amount for the debt, and therefore, a Yes, because at the time of the
foreclosure sale cannot be conducted in order making, both parties had capacity.
to answer for the unpaid debt. The foreclosure
sale is conducted upon Ds failure to pay P880, (RJPG: The term making means
000 is void since the amount demanded as the perfection of the donation; otherwise, if
outstanding loan was overstated, making means giving, Article 737 of the
consequently, it has not been shown that D Civil Code would be inconsistent with Article
has failed to pay his outstanding obligation. 734 which states that the donation is
perfected from the moment the donor knows
As a result, the subsequent of the acceptance by the done. Thus, at the
registration of the foreclosure sale cannot time the donation is perfected, both the donor
transfer any rights over the mortgaged and the done must be capacitated.)
property to C. The registration of the
foreclosure sale, which is void, cannot vest title 138. A Real Estate Mortgage contains a
over the mortgaged property. The torrens stipulation that the mortgagee shall send
system does not create or vest title where one notice of foreclosure proceedings to the
does not have a rightful claim over a real mortgagor at the latters given address. If the
property. It only confirms and records title mortgagee decides to extrajudicially foreclose
already existing and vested. It does not permit the mortgage, is personal notice to the
one to enrich himself at the expense of the mortgagor still necessary?
another. Thus, registration or real property by
one person in his or her name, whether by Yes. In extrajudicial foreclosure
mistake or fraud, the real owner being another proceedings, personal notice to the mortgagor
person, impresses upon the title acquired the is actually unnecessary unless stipulated. In
character of a constructive trust for the real this case, the mortgagor and the mortgagee
owner, which would justify an action for voluntarily agreed on an additional stipulation
reconveyance. Since the property has not yet embodied in their mortgage agreement. Not
been transferred by C to an innocent purchase being contrary to law, morals, good customs,
for value, D may still avail of the remedy of public order or public policy, the mortgagee
should send notice of the extrajudicial

BAR OPERATIONS 2013


I press toward the mark for the
prize of the high calling of God in
Christ Jesus. - Philippians 3:14

BARRISTERS CLUB Page 38 of 55


in CIVIL LAW

Prepared by: Atty. Roney Jone P. Gandeza

proceedings to the mortgagor in compliance


with the stipulation in the mortgage deed. The rules on redemption in case of an
(Tamayo, Jr. v. Heirs of Gavino Dominguez, 498 extrajudicial foreclosure of land acquired
SCRA 342 [2006]) under free patent or homestead statutes may
be summarized as follows: If the land is
QUESTIONS 139-141 are based on the mortgaged to parties other than rural banks,
following situation: the mortgagor may redeem the property
within one (1) year from the registration of the
In 1976, A obtained an original certificate of sale pursuant to Act No. 3135. If
certificate of title covering a parcel of land he fails to do so, he or his heirs may
secured through a homestead patent. Upon repurchase the property within five (5) years
As death in 1978, the land was transferred by from the expiration of the redemption period
succession to his son B who obtained a also pursuant to Section 119 of the Public Land
transfer certificate of title in his name. Act. Consequently, the complaint filed in 2002
In 1989, B mortgaged the land to DBP was on time.
as security for a loan. When B failed to pay,
the bank extrajudicially foreclosed the 141. If B is already deceased, may his
mortgage, purchased the property at the widow exercise the right of redemption?
public auction, and secured a title in its name
after consolidation. The term legal heirs as used in
Section 199 is used in generic sense. It is broad
Invoking Section 119 of the Public enough to cover any person who is called to
Land Act, B tried to repurchase the property the succession either by provision of a will or
in 2002, but the bank refused. The bank by operation of law. Thus, legal heirs include
defends that there can no longer be any right both testate and intestate heirs depending
of repurchase but by a transfer certificate of upon whether succession is by will of the
title, and that in any event, the right to testator or by law. Legal heirs are not
repurchase had already prescribed. necessarily compulsory heirs but they may be
so if the law reserves a legitime for them. The
139. Is B entitled to repurchase the subject interpretation of legal heirs is more in
property? keeping with the salutary purpose behind the
enactment of Section 119 and the
Yes, B is entitled to repurchase. The jurisprudence laid down on the matter.
plain intent of Section 110 of the Public Land Indeed, it is not far-fetched to arrive at a more
Act is to give a homesteader every chance to liberal conclusion if the section is analyzed in
preserve and keep in the family the land that accordance with its purpose.
the State has gratuitously given him as a
reward for his labor in cleaning, developing The widow inherited the property
and cultivating it. Hence, the fact that the land from B, her husband, who in turn inherited it
had been inherited by the patentees son (and from his father A. The widow, as daughter-in-
a new titled in his name is issued) does not law of the patentee, can be considered as
bring it outside the purview of Section 119. In among the legal heirs who can repurchase the
fact, the policy behind the law is fulfilled land. The Supreme Court has time and again
because the land remains in the family of the said that between two statutory
patentee. interpretations, that which better serves the
purpose of the law should prevail.
140. Is Bs right to repurchase already Furthermore, the law must be liberally
time-barred? construed in order to carry out its purpose.

BAR OPERATIONS 2013


I press toward the mark for the
prize of the high calling of God in
Christ Jesus. - Philippians 3:14

BARRISTERS CLUB Page 39 of 55


in CIVIL LAW

Prepared by: Atty. Roney Jone P. Gandeza

(DBP v. Gagarani, GR No. 172248, September


17, 2008) 144. What unions are covered by Articles
147 and 148 of the Family Code?
(RJPG: The term making must be Article 147 applies to two
interpreted to mean perfection of the relationships. The first is when a man and a
donation; otherwise, if making means woman who are capacitated to marry each
giving, Article 737 of the Civil Code would be other live exclusively as husband and wife
inconsistent with Article 734 which states that without the benefit of marriage. The second is
the donation is perfected from the moment when a man and a woman live together under
the donor knows of the acceptance by the a void marriage where the parties do not have
donee. Thus, at the time the donation is an existing marriage with another.
perfected, both the donor and the donee must
be capacitated.) Article 148, on the other hand, applies
to five kinds of relationships; namely (1)
142. Y, the prospective husband, gives a bigamous marriages; (2) adulterous
donation propter nuptias of a parcel of land relationships; (3) relationships in a state of
to X, his prospective wife. Due to quarrel, the concubinage; and (5) multiple alliances of the
marriage is called off. Is the donation same man.
automatically revoked?
145. X has no child. At the time he gave a
I distinguish. If the donation propter donation of P100, 000, he had P1 million.
nuptias is incorporated in X and Ys settlement, Therefore, after the donation, he had P900,
the donation is rendered void by the non- 000 left. Later he adopted a minor child. At
celebration of the marriage (Art. 81, Family the time he made the adoption, he had only
Code). However, if the donation is propter P50, 000 left. Should the donation be
nuptias is made independently of the marriage reduced? How much and within what period?
settlement, the donation is revocable only at The donation should be reduced by
the instance of the donor. (Art. 86 (1), Family P25, 000 because the legitime is impaired to
Code) that extent. The action to revoke or reduce the
inofficious donation must be brought within
143. Article 75 of the Family Code four years from adoption. Thus:
specifically provides that in the absence of a
marriage settlement, or when the regime 50, 00 (value at the time of adoption)
agreed upon is void, the system of absolute 100,000 (value of donation)
community of property shall govern the 150,000
property relations of the spouses during their
marriage. Is this rule absolute? The adopted child has the same rights
as a legitimate child. As such, he is entitled to a
The rule is not absolute. Pursuant to legitimate of P75, 000. But because the residue
Article 103 of the family Code, should the of Xs estate is only P50, 000, the donation is
surviving spouse contract a subsequent reducible by P25, 000.
marriage without liquidating the absolute
community of property regime or conjugal 146. Explain the doctrine of self-help in
partnership, a mandatory regime of complete civil law.
separation of property shall govern the
property relations of the subsequent marriage. The doctrine of self-help in civil law is
This is to protect the heirs of the deceased the right to counter in certain cases, forces
spouse. with force. It is enunciated under Article 429 of

BAR OPERATIONS 2013


I press toward the mark for the
prize of the high calling of God in
Christ Jesus. - Philippians 3:14

BARRISTERS CLUB Page 40 of 55


in CIVIL LAW

Prepared by: Atty. Roney Jone P. Gandeza

the Civil Code, and it can be exercised by the the owner may demand its separation, even
owner at the time of actual or threatened though the principal thing may suffer some
dispossession. When the possession has injury.
already been lost, the owner must resort to
judicial process for the recovery of property. (c) If the owner of the accessory thing has
made the incorporation in bad faith, he loses
147. Is the rule on accession discrete the thing incorporated and shall have the
absolute? obligation to indemnify the owner of the
The rule is not absolute. The following principal thing for the damages which the
are the instances when the owner has no right latter may have suffered.
to the fruits of his property:
(d) If the one who has acted in bad faith is
(a) If the thing is in the possession of a the owner of the principal thing, the owner of
possessor in good faith, in which case the accessory thing may choose between the
possessor is entitled to the fruits. (Art. 554, CC) former paying him its value or that his
accessory thing be separated, even though it
(b) If the thins is subject to a usufruct, in will cause damage or injury to the principal.
which case the usufructuary is entitled to the Moreover, the owner of the principal thing
fruits. (Art. 556, CC) shall be liable for damages.

(c) If the thing is leased, in which case the (e) If both owners had acted in bad faith,
lessee is entitled to the fruits leased, although their respective rights shall be determined as
such lessee must pay to the owner rentals though both had acted in good faith.
which are in the nature of civil fruits. (Art.
1654, CC) 149. What is meant by rebus sic stantibus?

(d) If the thing is in the possession of an Rebus sic stantibus is a rule which
antichretic creditor, in which case such provides that an agreement is valid only if the
creditor is entitled to the fruits with the same conditions prevailing at the time of
obligation of applying them to the interest and contracting continue to exist at the time of
principal. (Art. 2132, CC) performance.

148. What are the rules in adjunction? 150. What is the nature of reformation of
Adjunction is governed by the instruments as a remedy in law?
following rules:
Reformation of instruments (not
(a) If the two things can be separated reformation of contracts) is a remedy to
without injury, their respective owners may conform to the real intention of the parties
demand their separation. (Art. 469, par. 1 Civil due to mistake, fraud, inequitable conduct, or
Code) accident. The action prescribed in 10 years
from date of execution of instrument.
(b) If the two things cannot be separated
without injury, and both the owners had acted 151. What is an attractive nuisance?
in good faith, the owner of the principal thing
acquires the accessory indemnifying the owner An attractive nuisance is a dangerous
of such accessory for its value (Art. 456, Civil instrumentality or appliance which is likely to
Code). Nevertheless, if the accessory thing is attract children at play.
much more precious than the principal thing,

BAR OPERATIONS 2013


I press toward the mark for the
prize of the high calling of God in
Christ Jesus. - Philippians 3:14

BARRISTERS CLUB Page 41 of 55


in CIVIL LAW

Prepared by: Atty. Roney Jone P. Gandeza

152. What is the doctrine of attractive


nuisance? 155. In his will, testator T a) disinherits his
daughter, A, because she married a food for
One who maintains on his estate or nothing gigolo despite my repeated warnings
premises an attractive nuisance without that she shouldnt marry him; b) omits his
exercising due care to prevent children from wife, W; c) leaves a legacy of P10, 000 to his
playing therewith or resorting thereto, is liable mistress, M, and P5, 000 to his driver, E; and
to a child of tender years who is injured d) institutes his son, B, as his sole heir.
thereby, even if the child is technically a Distribute T;s estate of P100, 000.
trespasser in the premises. The reason for the
doctrine is that the attractiveness is an The disinheritance of A was ineffective
invitation to children. Safeguards to prevent because the ground relied upon by T does not
danger must be set up. constitute a valid ground for disinheritance
under Article 919 of the Civil Code. Hence, the
153. X was coerced into marrying Y. X sued testamentary provisions in the will shall be
for annulment. During the pendency of the annulled but only to the extent that As
case, X married Z. When Z learned of the first legitime was impaired.
marriage, Z sued X for bigamy. X now alleges
that the pendency of the annulment case is a The total omission of W does not
prejudicial question. Is X correct? constitute preterition because she is not a
compulsory heir in the direct line. Only
X is wrong because the decision in the compulsory heirs in the direct line may be the
annulment case is not important. The first subject of preterition. Not having been
marriage will either be annulled or not. If not preterited, she is entitled to her legitime.
annulled, bigamy can prosper. If annulled, still
bigamy can prosper, for when he married the The legacy in favor of M is void under
second time, he was still married to his wife, a Article 1028 of the Civil Code for being in
voidable marriage being considered valid until consideration of her adulterous relations with
annulled. T. She is, therefore, disqualified to receive the
legacy of P10, 000.
154. X, a married man, was forced by Y to
contract marriage with her. X then sued for The legacy of P5, 000 in favor of E is
annulment of the second marriage; Y not inofficious because it does not exceed the
retaliated with a charge of bigamy. In the free portion. Hence, E shall be entitled to
bigamy case, X moved to suspend the criminal receive it.
proceedings until after the termination of the
annulment case on the ground that the The institution of B, which applies only
annulment case is a prejudicial question. to the free portion, shall be respected.
Should the motion be granted?
In sum, the estate of T will be
Yes, because the annulment case distributed as follows:
poses a prejudicial question.
A 25, 000
If X was really forced into marrying Y, B 45, 000
then his consent was defective; hence, the W 25, 000
second marriage is to be annulled on that E 5, 000
ground. He cannot therefore be guilty of M 0
bigamy.

BAR OPERATIONS 2013


I press toward the mark for the
prize of the high calling of God in
Christ Jesus. - Philippians 3:14

BARRISTERS CLUB Page 42 of 55


in CIVIL LAW

Prepared by: Atty. Roney Jone P. Gandeza

156. Within what period may an action to presenting his case. (Mallion v. Alcantara, GR
revoke a donation inter vivos be filed by the No. 141528, October 31, 2006)
donor?
160. Articles 238 to 253 of the Family Code
An action to revoke a donation inter govern Summary Judicial Proceedings in the
vivos must be filed by the dono within four Family Law. Are these proceedings governed
years if the ground is the subsequent birth, by the Revised Rule on Summary Procedure?
adoption or reappearance of a child of the
donor or non-fulfillment of a condition; and it No. These summary judicial
must be filed within one year id the ground is proceedings fall within the jurisdiction of the
act of ingratitude in the part of the donee. As RTC, while the cases covered by the Revised
to the first two grounds, the right to file the Ruled on Summary Procedure are cases falling
action is transmitted to the heirs; as to the last within the jurisdiction of the Municipal Trial
ground only the donor can file the action and Court. These summary judicial proceedings are
the right is not transmitted to the heirs. in fact governed by the special rules in Articles
239 to 253 of the Family Code. Thus, in the
157. What is a muniment of title? proceedings to secure the consent of the
estrange spouse to a transaction, a preliminary
A muniment of title is an instrument or conference is to be conducted by the judge
written evidence which an application for land without the parties being assisted by counsel;
registration holds or possesses to enable him while under the Revised Rules on Summary
to substantiate and prove title to his estate. Procedure, the parties who are required to be
present in the preliminary conference, are
158. When is a notice lis pendens not assisted by counsel.
registrable or applicable?
161. A person who is a beneficiary in a will
A notice lis pendens is not registrable is competent to act as an instrumental
or applicable to the following: (a) preliminary witness in the execution of a will. Is this
attachment; (b) proceedings for the probate of statement accurate?
wills; (c) levies on execution; (d) proceedings
for administration of estate of deceased Yes, but his institution as an heir, or
persons; and (e) proceedings in which the only the legacy or devise given to him, shall be
object is the recovery of sum of money. rendered void, unless there are three other
competent witnesses (Art. 823, CC). In other
159. Does a previous final judgment words, he is disqualified from inheriting from
denying a petition for declaration of nullity of the testator (Art. 1027, CC).
marriage on the ground of psychological
incapacity bar a subsequent petition for 162. If a marriage is dissolved because of
annulment on the ground of lack of marriage the death of the husband, what surname may
license? the widow use?

Yes. In both petitions, the cause of Although the death of the husband
action is the same the declaration of nullity dissolves the marriage ties, still the window
of marriage. What differs is only the ground may desire to cherish her deceased husbands
upon which the case of action is predicated. A memory by the continued use of his surname.
party cannot avoid the application of res However, if she does not want to, she is
judicata by simply varying the form of his allowed to use her maiden surname. Notice
action or adopting a different method of

BAR OPERATIONS 2013


I press toward the mark for the
prize of the high calling of God in
Christ Jesus. - Philippians 3:14

BARRISTERS CLUB Page 43 of 55


in CIVIL LAW

Prepared by: Atty. Roney Jone P. Gandeza

the use of the word may in Article 373 of the 166. Suppose the insolvency of X is of
Civil Code. public knowledge but not known to D and the
creditor who also does not know, gives his
163. If a marriage is annulled, is the wife consent, may C hold D liable?
required to resume her maiden name and
surname? Yes, because the initiative came from
the debtor who should know the economic or
Wife is the guilty spouse: She SHALL financial status of his substitute or delegado.
resume her maiden name and surname.
167. If an agreement falling within the
Wife is the innocent spouse: She MAY Statute of Frauds is made in writing and duly
resume her maiden name and surname, but subscribed but the same is subsequently lost
she may choose to continue employing her or destroyed, is the agreement judicially
former husbands surname, unless (1) the enforceable?
court decrees otherwise; or (2) she or the
former husband is married again to another Yes, because the first (introductory)
person. (Art. 371, CC) part of the Statute of Frauds (Art. 1403, par. 2
Civil Code) states that if the agreement is not
164. What if legal separation occurs, is the in writing and duly subscribed, evidence of
wife entitled to continue using the husbands such agreement cannot be received without
surname even of she is the guilty spouse? the writing, or a secondary evidence of its
contents. Thus, the loss or destruction of a
Article 372 of the Civil Code does not written agreement (complying with the Statute
distinguish whether the wife is the guilty of Frauds) will not militate against the validity
spouse or not, unlike in the case of annulment or enforceability of the agreement. The
of marriage under Article 371, because in legal agreement us valid and enforceable and its
separation the marriage ties still subsist. existence and the contents thereof can be
proved by secondary evidence like the
165. D owes C P30, 000. D goes to C and testimony of the interested part and his
tells the latter that he is proposing X as the witness. (see Sec. 4, Rule 130, Rules of Court)
new debtor who will pay the debt. C accepts
the substitution. If X later becomes insolvent, 168. A orally sold to B his land. Later B
may C demand payment of the obligation wanted to have the land registered, but
from D? registration requires a public instrument. May
B compel A to execute the needed public
What obtains in this problem is instrument?
delegacion. In delegacion, the debtor offers,
and the creditor accepts, a third person who It depends. If the contract is still
consents to the substitution. Pursuant to executor, B cannot compel A to execute the
Article 1295 of the Civil Code, the insolvency of notarized sale because the contract is
X who has been proposed by D and accepted unenforceable. However, if the price has been
by C, shall not revive Cs action against D, the paid, or the land has been delivered, this time,
original obligor, except when said insolvency B can compel A because the contract is both
already existed and of public knowledge, or valid and enforceable.
known to the debtor when he delegated his
debt. QUESTIONS 169-170 are based on the
following fact situation:

BAR OPERATIONS 2013


I press toward the mark for the
prize of the high calling of God in
Christ Jesus. - Philippians 3:14

BARRISTERS CLUB Page 44 of 55


in CIVIL LAW

Prepared by: Atty. Roney Jone P. Gandeza

Among the items inherited by B from


his mother were some old paintings that had The statement in (B) is wrong because
been stored in his mothers attic for a number the fact that one of the parties to the contract
of years. B knew nothing about art and had has superior knowledge about the subject
no place to put them in his house. He placed matter of the contract does not by itself justify
an ad in the paper offering to sell the annulment, even of the other party is unaware
paintings at a price to be mutually agreed of that fact. Cs knowledge or lack of it was not
upon. C, a buyer for an art gallery, the principal cause on which the contract was
responded to the ad and examined the made and was not relied on by B in making the
paintings. From the signature and the style, C sale.
recognized that the artist was BenCab, a
renowned Filipino portrait artist. A and B The statement in (C) is wrong because
agreed upon a price and signed a contract. the fact that B was angry when he agreed to
the contract is not a ground for annulment of a
169. Which of the following facts, if true, contract under the law. Regardless of Bs state
would give B the best basis for avoiding the of mind, there was a meeting of the minds
contract with C? between the parties.

a. B told C that his mother had been The statement in (D) is incorrect
dabbled in painting when she was because Cs misrepresentations to B as to how
younger and had undoubtedly she will use the paintings does not appear to
painted them herself. go to the substance of the thing which is the
subject matter of the contract or to have been
b. B did not know that C was a buyer relied on by B. Hence, the misrepresentation is
for an art gallery and was very not significant enough to serve as a ground for
familiar with the works of renowned annulling the contract.
Filipino artists.
170. Which of the following facts, if true,
c. B told C that he wanted to get rid of would give C the best basis for annulling the
the paintings as soon as possible contract?
because he was angry at his mother
for giving away most of her a. Several of the paintings cracked
possessions to her friend just before when they were being transported by
he died. V because they were brittle with age.

d. C falsely told B that the paintings b. The day after the purchase, a
were to be used to furnish Cs newly- respected art historian announced in
constructed vacation house in a press release that several of Xs
Tagaytay City. paintings were actually done by his
students, causing the value of all Xs
The statement in (A) would enable B to paintings to decline.
annul the contract on the ground of mistake if
C was aware that B was mistaken about the c. Because of some experimental
identity of the artist. Under the facts in this pigments that the artist had used, the
choice, C knows that B is mistaken about the colors began to fade rapidly as soon
identity of the artist, which mistake refers to as the paintings were exposed to
the substance of the thing which is the object light; within a few days, virtually all of
of the contract. the colors had faded away.

BAR OPERATIONS 2013


I press toward the mark for the
prize of the high calling of God in
Christ Jesus. - Philippians 3:14

BARRISTERS CLUB Page 45 of 55


in CIVIL LAW

Prepared by: Atty. Roney Jone P. Gandeza

It depends. If the action is to enable W


d. The gallery for which C had to contract a subsequent marriage, the
procured the paintings was destroyed petition is open the estate of H to succession
by a fire shortly after the contract was or for other purpose, the petition will not
executed. prosper because the presumption is already
established by law.
(C) offers the best ground for annulling
the contract based on mutual mistake. When 172. Testator T has three legitimate
both parties entering into a contract are children, A, B and C. T made a will instituting
mistaken about facts relating to the his children, A and B, and a friend, F, as his
agreement, the contract may be avoidable by sole heirs. C was completely omitted in the
the adversely affected party if the latter did inheritance. If Ts estate at the time of his
not assume the risk or contingency of mistake. death is P90, 000, how should the distribution
Here, both parties believed that the paintings be made?
would be suitable for viewing and had no
reason to suspect that their color would fade The preterition of C annuls the
away when exposed to light. Despite Cs institution of A, B and F as Ts heirs. Intestacy
superior knowledge of the subject matter of results. Therefore, A, B and C will each get P30,
the contract, it is doubtful that she would be 000. The friend, F, get nothing.
deemed to have assumed the risk or
contingency of what occurred to the paintings. 173. Testator T has three legitimate
children: A, B and C. In his will, T disinherits A
and institutes B and C as his heirs. The
The circumstances in (A) are not that disinheritance of A is invalid because it is for a
strong as basis for annulment. Even assuming cause not provided by the law. If the
that both parties mistakenly believed that the hereditary estate is P90, 000, how shall the
paintings were not too fragile to be distribution be made?
transported, that risk is more likely to be
assumed by C. The institution of B and C remains
valid, but their shares are to be reduced to
The circumstances in (B) suggest only give A his legitime. Had there been preterion
that C has made a mistake as to the value of here, each would receive P30, 000 each.
the paintings, but since B knew nothing about Therefore, A is entitled to his legitime of P15,
the identify of the artist, the mistake is 000; B to his legitime of P15, 000 plus his share
unilateral and C cannot annual on this ground. as an instituted heir in the amount of P22, 500;
and C is entitled to the same share as B.
Choice (D) is incorrect because the
circumstances do not satisfy the requirements 174. X borrowed from his friend, Y, the
for annulment. Under Article 1351 of the Civil latters car for an out of town trip. On his way
Code, the particular motives of the parties in to his destination, X met with an accident
entering into the contract are different from which greatly damages the car. X was not at
the cause thereof. fault because he was driving carefully. If the
car is to be repaired, the expenses for repair
171. W, wife of H, institutes a petition seeking shall be borne by:
a judicial declaration of presumptive death of
her husband who has been missing since a. X alone, whether he is at fault or
2005. Will the petition prosper? not.

BAR OPERATIONS 2013


I press toward the mark for the
prize of the high calling of God in
Christ Jesus. - Philippians 3:14

BARRISTERS CLUB Page 46 of 55


in CIVIL LAW

Prepared by: Atty. Roney Jone P. Gandeza

b. Y alone, because X was not at fault. pledged. (Art. 2094, CC) If at all, there was a
pledge of the paper or document constituting
c. Y alone, he being the owner of the the Torrens Title, as a movable by itself, but
car. not of the land which the title represents.
There is no mortgage because no deed or
d. X any Y in equal shares, the contract was executed in the manner required
expenses being deemed by law for a mortgage. There is no contract of
extraordinary. antichresis because no right to the fruits of the
property was given to the creditor. (Art. 2131,
The choice in (D) is correct because the CC)
expenses are deemed ordinary, and not due to
the wear and tear of the thing. This is pursuant 177. A leased a building to B for a term of
to the second paragraph of Article 1949 of the ten years. B established a shoe factory in the
Civil Code which provides that if the extra- building and installed therein certain
ordinary expenses arise from the actual use of machineries. Are the machineries movables
the bailee of the thing loaned, even though he or immovables?
acted without fault, the expense is borne
equally by the bailor and the bailee share and The machineries are movables.
share alike. Machineries are deemed immovables only if:

175. A borrowed Bs truck. During a fire a) they are placed by the owner of the
which broke out in As garage, he had time to tenement.
save only one vehicle, and he saved his car
instead of the truck. Is A liable for the loss of b) an industry or works is carried on in
Bs truck? the tenement.

Yes. The baille in commodatum is c) machinery is intended for such


liable for the loss of the thing loaned, even if it industry or works.
should be through a furtuitious event, if being
able to save either the thing borrowed or his d) machinery tends directly to meet
own thing, he chose to save the latter. (Art. the needs of such industry or works.
1492, CC)
In the problem presented, the
176. D borrowed money from C. To machineries are movables since they were
guarantee payment, D left the Torrens title of installed by B, and not by A.
his land to C for the latter to hold until
payment of the loan. Is there a: 178. Suppose the lessee promised to leave
the machineries to the lessor at the end of
a) contract of pledge? the lease, would that make any difference in
your answer?
b) contract of mortgage?
Yes, because the machineries would
c) contract of antichresis? then be considered as immovables, because
the tenant would then be considered as the
d) none of the above? agent of the lessor.

None of the above. There is no pledge 179. A is the owner of a piece of land upon
because only movable property may be which fruits were grown, raised, harvested,

BAR OPERATIONS 2013


I press toward the mark for the
prize of the high calling of God in
Christ Jesus. - Philippians 3:14

BARRISTERS CLUB Page 47 of 55


in CIVIL LAW

Prepared by: Atty. Roney Jone P. Gandeza

and gathered by B in bad faith. Who owns the problem presented, the authority given by X to
fruits? Y was not in writing; hence, the sale is void.

A, the landowner, owns the fruits with 181. Paterno gave a power of attorney to
no obligation to indemnify B, except the Ambrosio authorizing him to sell Paternos car
latters expense in the production, gathering for Php200, 000 payable in cash. Under these
and preservation of the fruits. This is pursuant circumstances, what are the consequences of
to the rule that no one may unjustly enrich Ambrosios appointment by Paterno as the
himself at the expense of another. (Art. 442, latters agent to sell his car?
CC)
The authority of Ambrosio to sell the
ARTICLE 449: applies if the crops have car is express. It includes the implied authority
not yet been gathered (here the landowner to receive payment and to give a receipt as
gets the fruits without indemnity by the they are acts necessary to accomplish the
principle of accession continua) purpose of the agency. They are both actual
authority.
ARTICLE 443: applies when the crops
have already been gathered. Article 443 does If Paterno privately instructed
not apply when the planter is in good faith, Ambrosio not to consummate the sale, the
because under Article 544, he is already sale by Ambrosio is still binding upon Paterno
entitled to the fruits received and there is no as Ambrosio had apparent or ostensible
necessity of reimbursing him. authority to sell. As far as third persons are
concerned, the effect is as if Ambrosio had
180. X, who was abroad, phoned his actual authority.
brother, Y, authorizing him to sell Xs
property in Pasay. X sent the title to Y by The same is true if Paterno had not
courier service. Acting for his brother, Y authorized Ambrosio to sell the car but having
executed a notarized deed of absolute sale of knowledge that Ambrosio was acting for him
the land to Z after receiving payment. The kept silent and after consummation of the
sale is: sale, received the proceeds thereof from
Ambrosio. Here, Ambrosios authority rests on
a) valid, if the sale was notarized and estoppels on the part of Paterno to deny such
buyer paid in full. authority.

b) void, because X should have The authority given to Ambrosio to sell


authorized Y in writing to sell the land. the car is special because it involves a
particular transaction. Ambrosio has no
c) valid, because Y was his brother Xs authority to use the car for his own purposes
agent and entrusted with the title needed to but he can use it in an emergency as, for
effect the sale. example, to take a member of his family to a
hospital. In this case, his authority is
d) enforceable, because the buyer demanded by necessity.
could file an action to compel X to execute a
deed of sale. 182. Give examples of agency by estoppels
and implied agency.
(B) is correct because of the
requirement that the sale of realty through an AGENCY BY ESTOPPEL: P tells X that A
agent must be in writing to be valid. In the is authorized to sell certain merchandise. P

BAR OPERATIONS 2013


I press toward the mark for the
prize of the high calling of God in
Christ Jesus. - Philippians 3:14

BARRISTERS CLUB Page 48 of 55


in CIVIL LAW

Prepared by: Atty. Roney Jone P. Gandeza

privately instructs A not to consummate the liquidating its assets, Y and Z continued the
sale but merely to find out the highest price X business in the name of XYZ Ltd. Apparently
is willing to pay for the merchandise. If A without the objection of X. The withdrawal of
makes a sale to X, the sale is binding on P who X from the partnership was not published in
is in estoppels to deny As authority. In this the papers.
case, there is no agency created but there is a
power created in A to create contractual Is X liable for any obligation Y and Z
relations between P and a third person, might incur while doing business in the name
without having authority to do so. The legal of XYZ Ltd. after his withdrawal from the
result is the same as if A had authority to sell. partnership?

IMPLIED AGENCY: P authorized A to Yes, X can be held liable under the


sell his car. A sold the car to X who paid the doctrine of estoppels. But as regards the
purchase price. However, A did not give the parties among themselves, only Y and Z are
money to P. X is not liable to P. A has implied liable. X can be held liable since there was no
authority to receive payment. proper notification or publication to the public
in general. In the event that X is made to pay
183. Under Article 1772 of the Civil Code, the liability to a third person, he has the right
every contract of partnership having a to seek reimbursement from Y and Z. (Arts.
capital of 3, 000 or more, in money or 1837 to 1849, CC)
property, shall appear in a public instrument,
which must be recorded in the office of the 185. A, B and C are general partners in s
Securities and Exchange Commission. trading frim. Having contributed equal
Suppose this requirement has not been amounts to the capital, they agreed on equal
complied with, is the partnership still a distribution of whatever net profile is realized
juridical person, assuming that all other per fiscal period. After four years of
requisites are present? operation, A conveyed his whole interest in
the partnership to D without the knowledge
Yes. Article 1768 of the Civil Code and consent of B and C. Is the partnership
provides that the partnership has a juridical dissolved?
personality separate and distinct from that of
each of the partners, even in case of failure to No. Under Article 1813 of the Civil
comply with the requirements of Article 1772. Code, the conveyance by a partner of his or
her whole interest in the partnership does not
It is settled that the registration under itself dissolve the partnership on the absence
Article 1772 of the Civil Code is not intended as of an agreement.
a prerequisite for the acquisition of juridical
personality by the partnership, but merely as a 186. What are the rights of D shoule he
condition for the issuance of the license to desire to participate in the management of
engage in business or trade. the partnership and in the distribution of the
net profit of P45, 000 which was earned after
184. X, Y and Z entered into partnership his purchase of As interest?
under the firm name XYZ Ltd. Upon mutual
agreement, X withdrew from the partnership Also pursuant to Article 1813 of the
and the partnership was dissolved. However, Civil Code, D cannot interfere nor participate in
the remaining partners, Y and Z, did not the management or administration of the
terminate the business. Instead of winding up partnership business affairs. He may, however,
the business of the partnership and receive the net profits to which A would have

BAR OPERATIONS 2013


I press toward the mark for the
prize of the high calling of God in
Christ Jesus. - Philippians 3:14

BARRISTERS CLUB Page 49 of 55


in CIVIL LAW

Prepared by: Atty. Roney Jone P. Gandeza

otherwise been entitled. In This case, D is


entitled to 1/3 of the profits. Contrary to A and Bs allegation,
prescription has not even begun to run in the
187. Due to a misunderstanding, Atty. absence of a final accounting. Article 1842 of
Dumag withdraws his membership from a law the Civil Code states that the right to demand
firm which does not have a fized term. He accounting accrues at the date of dissolution in
later files with the SEC a petition for the the absence of any agreement to the contrary.
dissolution of the firm. After due proceedings, When a final accounting has been made, it is
the SEC hearing officer denied the petition on only then that prescription begins to run.
the ground that the withdrawal of Atty.
Dumag has not dissolved the firm In the case at bar, no final accounting
Is the SEC hearing officer correct? has been made, and that is precisely what the
heirs of C are seeking since A and B failed or
No. A partnership which does not have refused to render an accounting of the
a fixed term is a partnership at will. The law partnerships business and assets. Hence, the
firm Atty. Dumag is one such partnership. The action is not barred by prescription. (Emnace v.
birth and life of a partnership at will is Ca, 370 SCRA 431)
predicated on the mutual desire and consent
of the partners. 189. C loans D the amount of P100, 000. As
security for the loan, D delivered to C two
The right to choose with whom a rings by way of pledge. When D failed to pay,
person wishes to associate himself is the very C foreclosed, and had the rings sold at
foundation and essence of that partnership. Its auction. The proceeds of the sale, after
continued existence is, in turn, dependent on deducting expenses, amounted only to P70,
the constancy of that mutual resolve, along 000.
with each partners capability to give it. Verily,
anyone of the partners may, at his sole Is C entitled to demand the deficiency
pleasure, dictate dissolution of the partnership from D?
at will. He must, however, act in good faith. If
he acted in bad faith, it would not prevent the No. The Civil Code on pledge provides
dissolution of the partnership but that it can that the foreclosure of the pledge extinguishes
result in a liability for damages. (Ortega, et.al., the principal, whether or not the proceeds of
v. Court of Appeals, 245 SCRA 529 [1995]) the sale are more, or less than the amount of
the principal obligation. (Art. 2115, CC)
188. A, B and C are partners in a
construction firm. Due to a quarrel, C Under Article 2115 of the Civil Code, if
withdrew from the partnership in 1986 as a the price of the sale is less than the principal
result of which they agreed to dissolve their obligation, the creditor shall not be entitled to
partnership and executes an agreement of recover the deficiency, notwithstanding any
partition and distribution of the partnership stipulation to the contrary.
properties. In 1994, Cs heirs brought an
action against A and B for accounting of the 190. What if the rings were sold for the net
partnership assets and partition. A and B amount of P150, 000, is D entitled to the
defended on the ground of prescription. They excess?
contend that the action had already
prescribed four years after it accrued in 1986 Under Article 2115 of the Civil Code, if
when the partnership was dissolved by the the price of the sale is more than the amount
withdrawal of C. Has the action prescribed? of the principal obligation, the debtor shall not

BAR OPERATIONS 2013


I press toward the mark for the
prize of the high calling of God in
Christ Jesus. - Philippians 3:14

BARRISTERS CLUB Page 50 of 55


in CIVIL LAW

Prepared by: Atty. Roney Jone P. Gandeza

be entitled to the excess, unless it is otherwise The application on Article 1403, par.
agreed upon. (Art. 2115, CC) 2(e) of the Civil Code presupposes the
existence of a perfected contract of sale of real
191. Suppose the rings had been chattel property. It is a contractual grant, not of the
mortgaged to C, is D liable to pay the sale of real property involved but of the right
deficiency if the proceeds of the sale were of first refusal over the property sought to be
less than the indebtedness? Or, is she entitled sold. The right of first refusal need not be
to the excess, if the proceeds were more? written to be enforceable and may br proved
by oral evidence. (Rosencor Devt. V. Inquing,
If instead of a pledge the rings were 354 SCRA 119)
given by way of a chattel mortgage, C can
recover the deficiency from D as there is no 193. T died in 1990 with a will. In his will,
prohibition in the Chattel Mortgage Law he devised a house and lot to his friend, A, as
similar to pledge. The excess, if any, should be first heir and to C, As son, as second heir. B
returned to D, the mortgagor. died in 1995 survived by his two children E
and F. A himself died in 2000 survived by his
192. Lessor and Lessee verbally agree that children C and D.
if Lessor should decide to sell the building, he
will give Lessee the pre-emptive right to buy In the settlement of As estate, E and
the leased property. The following year, F filed a motion to exclude the house and lot
Lessor offered to sell to Lessee the building originating from T on the ground that they are
for P2 million, but Lessee counter-offered for the exclusive owners of the property. C and D
P1 million. opposed the motion on the ground that B, the
second heir, predeceased A, and that
Lessee later learned that a buyer had therefore, the fideicommissary substitution
already purchased the property from Lessor did not produce any effect as far as B, the
for P800, 000. He also discovered that the sale second heir, is concerned.
had already been consummated when Lessor
offered to sell it to him. Lessee thus offered Should the opposition be sustained?
to reimburse the buyer the purchase price of
P800, 000, plus an additional P200, 000 to No. B, the second heir, acquires a right
complete his earlier offer of P1 million. to the succession from the time of the
testators death, even though he, B, should die
When the offer was refused, Lessee before the fiduciary, A.
brought an action for rescission of the sale.
After due proceedings, however, the court B inherited from T as second heir when
dismissed the complaint on the ground that the latter died in 1990. When B died in 1995,
the right of redemption on which the he was able to transmit his right to his own
complaint is based is merely an oral one as heirs, E and F.
such, is unenforceable under the law.
When A (first heir) died in 2000, the
Is the right of refusal governed by the right of E and F over the property became
Statute of Frauds? absolute.

No. A right of first refusal is not among 194. H died leaving an estate of P100, 000.
those listed as unenforceable under the His widow, W, gave birth to a child four
Statute of Frauds. months after Hs death, but the child died five
hours after birth. Two days after the childs

BAR OPERATIONS 2013


I press toward the mark for the
prize of the high calling of God in
Christ Jesus. - Philippians 3:14

BARRISTERS CLUB Page 51 of 55


in CIVIL LAW

Prepared by: Atty. Roney Jone P. Gandeza

death, W also died because she had suffered Half to C; other half to G.
from difficult child birth. The estate of H is
now being claimed by his parents, A and B, F, the propositus, died with a will
and by C and D, the parents of W. Who is instituting his mother, E, as sole heir.
entitled to Hs estate of P100, 000?
Consequently, only one0half of the
If the child had an intra-uterine life of property passed to her by operation of law
not less than seven months, it inherited from since that is her legitime. Only this portion of
the father. Consequently, the estate of P100, the property has become reservable.
000 shall be divided equally between the child
and his mother as legal heirs. Upon the death B is not entitled to the reservable
of the child, its share of P50, 000 goes by portion of the property.
operation of law to the mother, W, which is
subject to reserve troncal. Although a relative of the propositus in
the second degree, B is merely related by
Under Article 891 of the Civil Code, the affinity to the ascendant (A) from whom the
reserve is in favor of relatives belonging to the property came. She does not, therefore,
paternal line and who are three degrees from belong to the line from which the property
the child. The parents of H (A and B) are came.
entitled to the reserved portion which is P50,
000 as they are two degrees related from the A reservatario must not only be
child. The P50, 000 inherited by W from H will related by consanguinity to the propositus
go to her parents, C and D, as her legal heirs. within the third degree, but he must also be
related by consanguinity to the ascendant
However, if the child had an intra- from whom the property came.
uterine life of less than seven months, half of
the estate of H, or P50, 000, will be inherited C is entitled to the reservable portion
by W, the widow while the other half, P50, of the property since he is not only a third
000, will be inherited by the parents of H. degree relative by consanguinity of the
Upon the death of W, her estate of P50, 000 propositus, but he also belongs to the line from
will be inherited by her own parents, C and D. which the property came.

195. Before his death in 1990, A donated G, on the other hand, is entitled to the
to his grandson, F, a child of his predeceased part of the property which is not reservable in
son D, a house and lot worth P600, 000. accordance with the ordinary rules of intestate
succession.
In 1995, F died with a will instituting 196. D, only daughter of B, married E, only
his mother, E, as his sole heir. His estate son of A, in 1981. A son, X, was born to the
consisted entirely of the house and lot which couple in 1982. E died in a vehicular accident
he had received from A. in 1984. In 1986, D married F, only son of C. A
son, Y was born to the couple in 1988. D also
In 1998, E also died but without a will. died in a vehicular accident in 1992. In 1995,
The house and lot is now claimed by: (a) B, X, who was very sickly, donated to his half
widow of A and grandmother of F; (b) C, son brother, Y, a parcel of land. X died the
of A and B uncle of F; and (c) G, sister and following year. In 1998 Y also died. He died
only living relative of E. To who, shall the intestate and without any surviving issue. The
property be adjudicated? land which he had acquired from X was
inherited by his father, F, who was his only

BAR OPERATIONS 2013


I press toward the mark for the
prize of the high calling of God in
Christ Jesus. - Philippians 3:14

BARRISTERS CLUB Page 52 of 55


in CIVIL LAW

Prepared by: Atty. Roney Jone P. Gandeza

legal heir. F died intestate in 2002, survived EXCEPT FOR CHATTEL MORTGAGE, a
only by his father, C. The land which had pledge, real estate mortgage, or antichresis
originated from X is now being claimed by A may exceptionally secure after-incurred
and B on the ground that it is reservable. C obligations so long as these future debts are
claims that the property belongs to him and accurately described. This is so because chattel
him alone as his inheritance from his son, F. mortgages can only cover obligations existing
at the time the mortgage is constituted.
Who among the grandfathers is
entitled to the property? One of the requirements of chattel
mortgage is an affidavit of good faith and the
B alone is entitled to the property. The law has provided that the parties to the
property is reservable. contract must execute an oath that the
mortgage is made for the purpose of securing
FIRST, the property had been acquired the obligation specified in the conditions
by operation of law by an ascendant (F)from thereof and for no other purposes. The debt
his descendant (Y) upon the death of the referred to in the law is a current obligation,
latter. not an obligation that is merely contemplated.

SECOND, the property had been 198. Which irregularities on formal


previously acquired by gratuitous title by the requisites of marriage would not affect the
descendant (Y) from a brother (X). validity of a marriage but may render the
party or parties civilly, criminally, or
THIRD, descendant (Y) died without administratively liable?
any legitime issue who can inherit from him.
a) Marriage license is issued by a local
In order to determine who can qualify civil registrar of a city or municipality where
as a reservatario, two test should be applied: neither party habitually resides.
First is the claimant a relative of the
descendant-propositus within the third b) Marriage license is pre-signed by a
degree? Second, does he belong to the line local civil registrar but issued by a fixer.
from which the reservable property came?
c) Marriage is performed at a place
Applying the tests, it is clear that: other than those designated by law without
the written request under oath of the parties.
Neither can C qualify because he does
not belong to the line from which the property d) No marriage counseling certificate is
came. He is not related by consanguinity to X. attached to the application and the marriage
license is issued even before the lapse of the
Only B can qualify. He is not a relative three0month period, the marriage is valid.
of Y within the third degree; he also belongs to
the line from which the reservable property e) Marriage ceremony is performed
came. with only one witness present or with no
witnesses at all (secret marriage).
197. May a contract of pledge, real estate
mortgage, antichresis and chattle mortgage f) Marriage license is issued without
secure after-incurred obligation? waiting for the lapse of the three-month
period.

BAR OPERATIONS 2013


I press toward the mark for the
prize of the high calling of God in
Christ Jesus. - Philippians 3:14

BARRISTERS CLUB Page 53 of 55


in CIVIL LAW

Prepared by: Atty. Roney Jone P. Gandeza

g) Marriage license is issued without 201. A and B are childhood friends. Because
waiting for the lapse of the ten-day period, or of their friendship, A gave to B in usufruct a
if the notice of application is not published. parcel of land to last up to the time their high
school teacher, Mrs. X, reaches the age of 70.
Mrs X died at the age of 65. Is the usufruct
h) Marriage remains valid neither extinguished?
contracting party belongs to the solemnizing
officers church or religious sect. No. Under Article 606 of the Civil Code,
a usufruct granted for a time that may elapse
i) The solemnizing officer is not duly before a third person attains a certain age shall
registered with the civil registrar general at the subsist from the number of years specified
time of the marriage. even if the third person should die before the
period expires, unless such usufruct has been
j) The solemnizing officer solemnized granted only in consideration of the existence
the marriage in violation of the written of such person.
authority granted to him by his church or
religious sect, the marriage is valid. If the document constituting V as a
usufructuary does not state that it will end the
k) A judge solemnizes a marriage moment Mrs. X is dead, then it will continue.
outside his courts jurisdiction.

l) Alien spouse obtains a marriage


license without a certificate of legal capacity.

199. If a party has been previously


married, and the prior marriage has been
dissolved, is parental consent still required if
such party is still below 21 years of age?

Not anymore. This is so because of


Article 14 of the Family Code which provides,
In case either or both of the contract parties,
not having been previously emancipated by a
previous marriage, x x x. This means parental
consent is no longer required for these parties.

200. What is a mortgage in possession?

A mortgage in possession, otherwise


known as antichresis, is one where the
mortgagee acquires actual or constructive
possession of the property mortgaged for
purposes only of enforcing his security over
the property and collecting the income to pay
for the mortgagors debt. (Nadal v. CA, 320
SCRA 699)

BAR OPERATIONS 2013


I press toward the mark for the
prize of the high calling of God in
Christ Jesus. - Philippians 3:14

BARRISTERS CLUB Page 54 of 55


in CIVIL LAW

Prepared by: Atty. Roney Jone P. Gandeza

THE BARRISTERS CLUB OFFICERS


Chancellor: ABBYGAILE T. GONZALES
Vice Chancellor: ROMEL L. BASILAN
Secretary: JESSA ALYSSA G. REYES
Treasurer: MILDRED P. AMBROS
PRO: ROBYN B. DELA PENA
PRO: AARON JAMES E. CO
Business Manager: RUDDY ALLEN N. YEE
Business Manager: LESLIE D. RAGUINDIN
SSG Representative: ANNE LUCILLE B. RUIZ
Ex-Officio: RONA B. ESTRADA
Adviser: ATTY. ISAGANI G. CALDERON
Dean, College of Law: ATTY. REYNALDO U. AGRANZAMENDEZ

BAR OPERATIONS 2013


I press toward the mark for the
prize of the high calling of God in
Christ Jesus. - Philippians 3:14

BARRISTERS CLUB Page 55 of 55

Vous aimerez peut-être aussi